Da war das Universum noch nicht mal 1 Milliarde Jahre alt (also verglichen mit einem Menschenleben quasi noch ein Kindergartenkind) und war trotzdem schon von 30 Millionen schwarzen Löchern bevölkert. Das zumindest zeigen die Ergebnisse, die das Weltraumteleskop Chandra kürzlich geliefert hat.

Chandra beobachtet den Himmel im Röntgenlicht und das ideal für die Suche nach schwarzen Löchern. Denn wenn die sich Gas und Staub einverleiben, entsteht dabei jede Menge Röntgenstrahlung und auch wenn man sie ja sonst eigentlich aus Prinzip nicht sehen kann, strahlen sie (oder besser: ihre Umgebung) im Röntgenlicht enorm Himmel.

Chandra hat das gemacht, was sein berühmterer Teleskopkollege Hubble früher schon gemacht hat: eine “Deep-Field-Aufnahme”. Beim Hubble Deep Field. Da hat Hubble tagelang das selbe Stück scheinbar leeren Sternehimmels betrachtet und Licht gesammelt. Am Ende zeigte sich: der Himmel war gar nicht leer sondern voller Galaxien die enorm weit entfernt und deswegen enorm schwach sichtbar waren. Weit weg bedeutet in der Astronomie auch immer: sehr alt. Hubble hatte mit seinem Deep Field einen faszinierenden Blick ins frühe Universum getan.

i-99e13830eb1331d96652f38effc4d5f3-cdfs_chandraB-thumb-500x323.jpg
Das südliche Chandra Deep Field: links die Röntgenaufnahme; jeder Punkt ist eine helle Röntgenquelle. Rechts ist ein Ausschnitt des Deep Fields mit einer Aufnahme des Hubble-Teleskops im optischen Bereich überlagert (Bild: X-ray: NASA/CXC/U.Hawaii/E.Treister et al; Optical: NASA/STScI/S.Beckwith et)

Chandra hat nun sechs Wochen lang Röntgenstrahlung von ein und der selben Region des Himmels gesammelt. So war es möglich herauszufinden, ob die Galaxien im frühen Universum auch schwarze Löcher in ihren Zentren hatten so wie das heute bei den großen Galaxien (wie unserer Milchstrasse) der Fall ist. Die Interaktion zwischen Galaxien und ihren schwarzen Löchern ist ja sehr komplex. Und über die schwarzen Löcher in der Frühzeit des Universums wusste man noch gar nichts. Gab es damals überhaupt welche? Wenn ja, waren sie genauso groß wie die heutigen? Wuchsen sie schneller oder langsamer?

Jetzt hat man endlich ein paar Antworten. Ja, es gab schwarze Löcher! Auf den Aufnahmen des Chandra Deep Field konnte etwa 200 Galaxien finden die 800 bis 950 Millionen Jahre nach dem Urknall entstanden sind untersuchen. Mindestens 30 Prozent davon – und vielleicht sogar alle – enthalten schnell wachsende supermassive schwarze Löcher. Ok, 30% von 200 sind nicht viel. Aber Chandra hat ja auch nur einen kleinen Ausschnitt des Himmels beobachtet. Rechnet man das auf den ganzen Himmel hoch, dann kommt man auf mindestens 30 Millionen schwarzer Löcher, die die frühen Galaxien bevölkert haben. Diese Erkenntnisse sind wichtig, wenn man die Entwicklung der Galaxien genauer verstehen will. Schon damals haben sich also die Galaxien gemeinsam mit ihren schwarzen Löchern entwickelt und beide haben sich gegenseitig beeinflusst…

Kommentare (144)

  1. #1 nix
    16. Juni 2011

    .

  2. #2 nihil jie
    16. Juni 2011

    @FF

    Diese Erkenntnisse sind wichtig, wenn man die Entwicklung der Galaxien genauer verstehen will. Schon damals haben sich also die Galaxien gemeinsam mit ihren schwarzen Löchern entwickelt und beide haben sich gegenseitig beeinflusst…

    Aber das hatte man schon länger vermutet… so weit ich noch auf dem Laufenden bin. Wer war das noch mal der schon seit Jahren darüber forscht…. Vera Rubin oder Andrea Ghez ???

  3. #3 Soziobloge
    16. Juni 2011

    Da die Astronomen ja quasi immer in die Vergangenheit schauen. Kann man eigentlich sagen, wie das Universum zur Zeit aktuell aussieht? Zumindest bestimmte Parameter vielleicht?

  4. #4 nihil jie
    16. Juni 2011

    @Soziobloge

    ja klar…. Groß, Kalt und Dunkel 😉

  5. #5 Alderamin
    16. Juni 2011

    Da die Astronomen ja quasi immer in die Vergangenheit schauen. Kann man eigentlich sagen, wie das Universum zur Zeit aktuell aussieht? Zumindest bestimmte Parameter vielleicht?

    Na, so wie bei uns in der Gegend, halt. Man geht ja zurecht davon aus, dass sich das Universum überall ungefähr gleich entwickelt hat. D.h. “da” sieht es jetzt genau so aus wie “hier”, wobei “hier” schon ein paar hundert Millionen Lichtjahre umfasst. In den paar hundert Millionen Jahren, die das Licht von dort bis zur Erde gebraucht hat, hat sich nicht mehr viel getan. Die meisten Veränderungen fanden in den ersten paar hundert Millionen Jahren nach dem Urknall statt, als die ersten Sterne und Galaxien entstanden, und die Galaxien dann zu größeren miteinander verschmolzen.

    Und wie sieht es hier aus? Gruppen von Galaxien, die sich in größeren Haufen organisieren, die wiederum Superhaufen entlang von Filamenten bilden, zwischen denen große Leere herrscht. Und in den Galaxien: Gas, Staub, Sterne, die aus dem Material vorangegangener Sterne bestehen und zum großen Teil von Planeten umkreist werden, sowie Überreste verloschener Sterne (weiße Zwerge, Neutronensterne, schwarze Löcher). Und ein supermassives Loch im Zentrum einjeder Galaxie.

  6. #6 Alderamin
    16. Juni 2011

    “Und ein supermassives schwarzes Loch im Zentrum einjeder Galaxie.” muss es natürlich heißen.

  7. #7 BlibberOlaf
    16. Juni 2011

    Im Buch “Kosmologische Strukturbildung: Von der Quantenfluktuation zur Galaxie”
    https://www.amazon.de/Kosmologische-Strukturbildung-Quantenfluktuation-Galaxie-Astrophysik/dp/3827419352

    heißt es auf Seite 16 das die dunkle Materie zu erst Schwarze Löcher ausbildete, da sie nach unserem Kenntnisstand nur mit der Gravitationskraft und nicht mit den anderen 3 uns bekannten Kräften wechselwirkt.
    Die dunkle Materie bildete damit die Geburtstätten unserer Galaxien, die ja aus baryonischer Materie bestehen —

  8. #8 Alderamin
    16. Juni 2011

    Hmm, ich hab’ mal gehört, dass dunkle Materie keine massiven Klumpen bilden kann (und somit auch keine schwarzen Löcher), eben weil sie nicht anders als durch Gravitation wechselwirken kann. Wie sollen die Teilchen der dunklen Materie ihre kinetische Energie in Form von Wärme abgeben, wie das z.B. bei der Enstehung von Sternen aus Akkretionsscheiben geschieht, wenn sie keine mechanische Reibung kennen?

    Deswegen soll etwa die Milchstraße ein Halo aus dunkler Materie umgeben, und diese nicht in der Milchstraßenscheibe konzentriert sein.

    Auf jeden Fall gibt es aber irgendeinen, noch nicht verstandenen Prozess, der die Masse des zentralen schwarzen Lochs mit der Größe des kugelförmigen Zentralteils (bulge) der zugehörigen Galaxie korreliert. Das wäre im verkleinerten Maßstab in etwa so, als ob ein Sandkorn die Größe eines Objekts von den Ausmaßen der USA steuern könnte.

  9. #9 Eris Baran
    16. Juni 2011

    @Soziobloge

    Würde mich auch sehr interessieren,insbesondere, ob es einen “Jetzt” -Zustand gibt, der überall im Universum gleichzeitig als eben dieser “jetzige Moment” wahrgenommen oder als solche bezeichnet werden kann.
    Das “Jetzt” ist ja ein “zeitlicher” Begriff ,wobei die Zeit in der Physik an den Raum gekoppelt und mit einer Richtung (vor/zurück) versehen wird.
    Meine schwachformulierte Frage lautet: Ist das “Jetzt” auch ein relativistisches Phänomen, das unter unterschiedlichen Raum/Ort- und Geschwindigkeitverhältnissen etc. die exotischsten Emanationen eruiert.
    Anders gefragt: Was geschieht mit den “Jetzigkeiten” bzw.”Gleichzeitigkeiten” beim Ritt von Albert (Einstein) auf einem Lichtstrahl und seiner vergreisenden Fangemeinde auf der guten Alten Erde?Ist unser “Jetzt” auch sein “Jetzt”?Gibt es ohne Raum auch kein “Jetzt” (z,B, bei der monströsen Singularität).Ist mein “Jetzt” auch das “Jetzt” am Rande des Universums?Ist das “Jetzt” überhaupt physikalisch formulierbar.
    Ist das “jetzt” schlicht und einfach “Jetzt”und immer wie ewig das Jetzt, überall, alles durchdringend ……?Wäre das “Jetzt” ohne Raum und Materie überhaupt wahrnehmbar? (Blöde Frage , ich weiss ,da auch dann nichts wäre, was wahrnehmen könnte.)Gibt es letztendlich ein “objektives Jetzt” ,das fern von Relationen stattfindet…oder ist es reines Phänomen der Wahrnehmung,
    Tut mir Leid, ich habe diese Woche Nachtschicht gearbeitet, fühle michsomit etwas umnachtet…
    Gruss an alle

  10. #10 Florian Freistetter
    16. Juni 2011

    @Eris Baran: “Würde mich auch sehr interessieren,insbesondere, ob es einen “Jetzt” -Zustand gibt, der überall im Universum gleichzeitig als eben dieser “jetzige Moment” wahrgenommen oder als solche bezeichnet werden kann”

    Seufz.. irgendwann muss ich da wohl mal einen eigenen Artikel drüber schreiben.

    Einstein hat seinen berühmten Aufsatz aus dem die spezielle Relativitätstheorie entstand geschrieben:

    Wir haben zu berücksichtigen, dass alle unsere Urteile, in welchen die Zeit eine Rolle spielt, immer Urteile über gleichzeitige Ereignisse sind. Wenn ich zum Beispiel sage: “Jener Zug kommt um 7 Uhr an.” So heißt dies etwa: “Das Zeigen des kleinen Zeigers auf sieben und das Ankommen des Zuges sind gleichzeitige Ereignisse.” Es könnte scheinen, dass alle die Definition der Zeit betreffenden Schwierigkeiten dadurch überwunden werden könnten, wenn ich an Stelle von “Zeit” die “Stellung des kleinen Zeigers meiner Uhr” setze. Eine solche Definition genügt in der Tat, wenn es sich darum handelt, eine Zeit zu definieren ausschließlich für den Ort, an welchem eben sich die Uhr befindet. Die Definition genügt nicht mehr, sobald es sich darum handelt, an verschiedenen Orten stattfindende Ereignisse miteinander zu verknüpfen, oder – was auf das selbe hinausläuft – Ereignisse zeitlich zu werten, welche in von der Uhr entfernten Orten stattfinden. –

    Darüber zu spekulieren wie es “Jetzt” irgendwo anders aussieht, ist relativ müssig. Wir können über dieses “Jetzt” nichts in Erfahrung bringen, prinzipiell nicht.

  11. #11 Eris Baran
    16. Juni 2011

    @Florian Freistetter:Darüber zu spekulieren wie es “Jetzt” irgendwo anders aussieht, ist relativ müssig. Wir können über dieses “Jetzt” nichts in Erfahrung bringen, prinzipiell nicht.

    Hallo Florian! Darum geht es mir im Grunde auch nicht.Also, nicht darum wie es jetzt woanders a u s sie h t , sondern wie das “Jetzt” zu verstehen oder zu begreifen ist.
    Nicht unbedingt die Gleichzeitigkeit von Ereignissen , die der olle Albert im obigen Zitat relativiert, sondern , ob es mit Ereignisssen oder auch schlicht ereignislos einen uberall gültigen jetzigen Moment gibt, der vielleicht auch in den 500 Milliarden (?) Paralleluniversen von Hawking gerade eben dieser jetzige Moment ist, d.h von Orten Räumen ,Ereignissen,Beobachtern kausal unabhängig “Jetzt”-Die Frage ist auch nicht metaphysisch gemeint, obwohl mir dazu einige buddhitishe Thesen einfallen……, sie ist alltäglicher Natur und dennoch irgendwie numinos.

    Wahrscheinlich waren meine Fragen weiter oben etwas schwerfällig formuliert.
    Zumindest ging es mir nicht darum wie etwas , wann und wo ankommt und gemessen
    wird, um Relationen herzustellen, sondern ob es für all diese Faktoren einen steten Fluss an gemeinsamen
    Momenten gibt , den wir als das “Jetzt” wahrnehemn.(Auch ohne Beobachter oder einen
    wahrnehmen sollte es doch dieses Jetzt immer geben, nicht?)

    Und dann noch das Jetzt aus relativistischer Sicht: Albert auf dem Lichtstrahl , ich auf der Erde……..Ist Alberts “Jetzt” gestreckt und länger und mein Jetzt kürzer ?
    Es kann ja nicht mit der Wahrnehmung allein erklärt werden, da einer von uns tatsächlich schneller altert…!Und ein ruhender Dritter, der direkt und ohne Zeit-und Informationsverlust mit mir und Albert verbunden ist, sagt :”Jetzt”, so dass Albert und ich und auch jener Dritte dieses Ausruf zeitgleich wahrnehmen….wir würden alle gemensam diesen einen Moment als ein “Jetzt” erfahren., nicht?………Ich hab das Gefühl, ich sollte mich endlich schlafen legen…..hmmm…ich hoffe, ich mach mich hier nicht lächerlich

  12. #12 Alderamin
    16. Juni 2011

    @Eric Baran

    Es gibt zwar für jeden Beobachter ein “Jetzt”, aber verschiedene Beobachter, die sich zueinander bewegen, haben nicht dasselbe “Jetzt”.

    Lies’ mal hier, das beantwortet schon einiges.

  13. #13 frantischek
    16. Juni 2011

    Kann man sich sicher sein das jede in der Aufnahme gefundene Röntgenquelle ein schwarzes Loch ist?

  14. #14 nihil jie
    16. Juni 2011

    Was mich aber wirklich interessieren würde aber was wir alle nicht erfahren werden ist, wie sich die jungen Galaxien so entwickelt haben bis heute…. also dem unserem heute. Oder Quasare… was ist aus denen in den Jahrmilliarden geworden. Kam mir gerade so in den Sinn,nach dem hier so viel über das “Jetzt” geschrieben wurde 😉

    Da ich endlich heute mehr Zeit habe, habe ich mal nachgeschaut wer sich da so intensiv mit den galaktischen massiven SL beschäftigt hat… Andrea Ghez war es. Ich kann mich aus irgend einer Doku über galaktische massive SL erinnern, dass sie im Interview sinngemäß meinte, sie wäre nicht überrascht wenn es in Zentrum jeder Galaxie ein SL säße… wohl schon immer von Anfang an. Und das muss so gegen ende der 90er gewesen sein. Wie auch immer… da scheint es nahe zu so eine Art “symbiotischer” Verbindung zwischen den massiven zentralen SL und den Galaxien die sie beinhalten, geben. Die wohl zerstörerischste und gefräßigste Struktur im Universum ist maßstäblich an den strukturierungs und dynamischen Prozessen von Galaxien beteiligt.

  15. #15 nihil jie
    16. Juni 2011

    Nachtrag:

    Ich habe vorhin mal so bisschen auf der Chandra Webseite herumgestübert 🙂 Bin eine weile auch bei den technischen Daten gelandet… Da haben sie ein echtes coole Stück Elektronik und Technik, in dem Kasten, verbaut… meine Herrn *g* Schon nicht zu verachten. Na ja… aber im Grunde können die Dinger nie gut genug sein. Eigentlich bräuchte man noch etwas womit sich Neutrinos besser detektieren lassen, und natürlich Gravitationswellen. Was sich uns da für ein Bild des Universums auftun würde… kann ich mir gerade gar nicht so richtig ausmalen 😉

  16. #16 Weltraumeule
    16. Juni 2011

    Wurde gerade von dem Kommentar eines anderen Benutzers irritiert. Jener meinte, nach dem Urknall, bei der Expansion des Universums, hätte sich das Universum bzw. der Raum schneller als das Licht ausgedehnt. Ist das in dem Falle möglich? Also dass das Licht und alles nur innerhalb des Raums auf seine Geschwindigkeit begrenzt ist, nicht jedoch der Raum selber?

  17. #17 Alderamin
    16. Juni 2011

    @mihiel jie

    Was mich aber wirklich interessieren würde aber was wir alle nicht erfahren werden ist, wie sich die jungen Galaxien so entwickelt haben bis heute…. also dem unserem heute. Oder Quasare… was ist aus denen in den Jahrmilliarden geworden.

    Die Galaxien waren früher wesentlich kleiner als heute. Man geht davon aus, dass sich viele kleine zu größeren Galaxien vereinigt haben. Auch unsere Milchstraße hat sich schon einige Zwerggalaxien einverleibt. Aus vielen kleinen Galaxien wurden wenige große.

    Quasare sind die schwarzen Löcher in den Zentren der Galaxien, wenn sie Materie schlucken. Das Gas sammelt sich vor dem Absturz in einer Scheibe um das schwarze Loch, die stark verdichtet und aufgeheizt wird, und dann in allen Frequenzen bis hin zur Röntgenstrahlung leuchtet. Wenn der Quasar irgendwann keinen Nachschub an Material mehr erhält, erlischt er. Dann wird er zu einem stillen schwarzen Loch, wie das in der Milchstraße.

  18. #18 Alderamin
    16. Juni 2011

    @Weltraumeule

    Ja, das ist möglich. Die Relativitätstheorie verbietet nur, dass sich etwas im Raum schneller als das Licht bewegt. Was der Raum selbst tut, ist eine andere Sache. Nicht nur während des Urknalls, auch jetzt weichen Galaxien, die sehr weit von uns entfernt sind, scheinbar mit mehr als Lichtgeschwindigkeit zurück.

    Eigentlich ist es aber so, dass sowohl diese Galaxien, als auch die unsere, praktisch still im Raum steht. Wenn wir die Geschwindigkeit der Milchstraße daran messen, wie der kosmische Hintergrund in verschiedenen Himmelsrichtungen rotverschoben ist (Dopplereffekt), dann kommen nur ein paar hundert km/s zusammen, mit denen die Milchstraße und ihre Nachbargalaxien umeinander tanzen. Das gleiche würde ein Beobachter in der fernen Galaxie messen. Nur der Raum dazwischen, der wächst ganz langsam, es kommt neuer Raum dazu und dadurch entfernen sich die Galaxien voneinander und Lichtwellenlängen werden auf dem Weg zu uns gedehnt, so dass es den Anschein hat, die Galaxie bewege sich sehr schnell im Raum, obwohl sie stillsteht. Wenn eine Galaxie so weit weg ist, dass sie sich mit mehr als Lichtgeschwindigkeit zu entfernen scheint, hat das Licht keine Chance, von dort zu uns zu dringen, weil der Raum schneller zunimmt, als das Licht in der gleichen Zeit an Strecke zurücklegt. Und so könnten wir solche Galaxien niemals sehen.

  19. #19 nihil jie
    16. Juni 2011

    @Alderamin

    Ja stimmt… den Vorgang ist ja auch schon so erwähnt und beschreiben worden. Ich frage mich manchmal aber, ob es all den kompakten Objekten so ergangen ist. Wurden sie alle zum Zentren einer Galaxie. Aber eigentlich müsste es… denke ich.

  20. #20 Wurgl
    16. Juni 2011

    Zu dem Thema passend, aber eben nicht gar so weit weg
    https://www.spiegel.de/wissenschaft/weltall/0,1518,768817,00.html

  21. #21 BuckRogers
    16. Juni 2011

    So. Nach etlichen Stunden des Lesens aller möglicher Artikel und der vielen Kommentare auf dieser Spitzenseite gebe ich heute auch mal meinen Senf dazu.
    Ich bin zwar “nur” Laie, aber ich sauge alles an Infos auf was ich so kriegen kann, auch wenn ich nicht immer alles verstehe. Teils kommen mir Fragen ins Bewusstsein, die wohl aus meinem Mangel an Fachwissen herrühren, aber man gibt sich halt immer Mühe eine Art von Verständnis zu entwickeln, für das man nicht an der Uni studiert haben muss. Dafür ist scienceblog.de auch sehr hilfreich:) Aber diese Frage “Da die Astronomen ja quasi immer in die Vergangenheit schauen. Kann man eigentlich sagen, wie das Universum zur Zeit aktuell aussieht?” habe ich mir auch schon soo oft gestellt. Erst recht wenn ich Bilder/Animationen zur Materieverteilung im All ansehe. All die Superhaufen und die große Mauer und und und, aber all das ist strenggenommen die Vergangenheit, obwohl das Licht erst JETZT mit uns wechselwirkt. Klar, es ist die Natur der Dinge, es wird einem immer verschlossen bleiben, wie es JETZT dort aussieht, wovon wir nur das sehen was z.B vor 5 Milliarden Jahren war. Ich kann mich aber nicht gegen die Frage wehren, wie es wäre ein Photon zu sein und 5 Milliarden Lichtjahre in diese Richtung zu fliegen, um mir dabei im Zeitraffer die Entwicklung anzusehen, wobei für mich die Zeit steht.
    Ich weiß, das ist Mumpitz, aber ich bin ja nicht der einzige der sich solche Fragen stellt.
    Und es macht ja auch Spass:)
    Aber die Zeit kann einem schon den Kopf verdrehen; wenn z.B. gesagt wird, dass es 400 mio Jahre dauerte bis die ersten Sterne entstanden, will ich nicht so recht begreifen, wie man diese Zeit definieren kann, wenn Zeit doch so relativ ist und es vorher ja noch keine Sterne gab an die man sich halten kann.
    So. Ich hoffe ihr mögt Senf:)

  22. #22 nihil jie
    16. Juni 2011

    @BuckRogers

    Da schreibe ich mal als erster willkommen im Blog 😉

    So. Ich hoffe ihr mögt Senf:)

    ach doch… also ich schon 😉

  23. #23 Alderamin
    16. Juni 2011

    Ich kann mich aber nicht gegen die Frage wehren, wie es wäre ein Photon zu sein und 5 Milliarden Lichtjahre in diese Richtung zu fliegen, um mir dabei im Zeitraffer die Entwicklung anzusehen, wobei für mich die Zeit steht.

    Nach der Relativitätstheorie wäre aus der Sicht eines Photons der Weltraum in Bewegungsrichtung unendlich dünn zusammengestaucht, es wäre aus seiner Sicht auch bei 5 Milliarden Lichtjahren zurückgelegter Entfernung sofort am Ziel. Wenn Du die Welt im Zeitraffer sehen wolltest, müsstest Du ein wenig langsamer als ein Photon unterwegs sein. Lichtgeschwindigkeit ist aber ohnehin nur masselosen Teilchen vorbehalten.

    wenn z.B. gesagt wird, dass es 400 mio Jahre dauerte bis die ersten Sterne entstanden, will ich nicht so recht begreifen, wie man diese Zeit definieren kann, wenn Zeit doch so relativ ist und es vorher ja noch keine Sterne gab an die man sich halten kann.

    Man kann ja aus der Galaxienflucht rückwärts auf den Zeitpunkt zurückrechnen, als alles an einem Punkt losging. Das ist der Nullpunkt der Zeitrechnung, der Urknall. Und von dem aus gerechnet 400 Millionen Jahre später entstanden dann eben diese Sterne. Wann ein Objekt, das man heute beobachtet, existiert hat, schließt man aus seiner Rotverschiebung z. Der Urknall läge bei unendlicher Rotverschiebung, den kann man nicht sehen. Das erste, was man sieht, ist die Strahlung nach der Rekombination von Elektronen und Kernen; vorher war das Weltall zu heiß für gebundene Elektronen, sie bewegten sich frei als Plasma, und ein Plasma ist für Licht undurchlässig, da es jede Wellenlänge absorbieren kann. Diese Hintergrundstrahlung des einst 3000 K heißen Plasmas ist mit z=1000 auf eine Temperatur von nur noch 3 K gestreckt und entspricht (das kann man ausrechnen) einem Weltalter von 380000 Jahren. Alles, was näher ist, hat ein geringeres z, und jedem gemessenen z entspricht genau ein zugehöriges Weltalter. Welches, das hängt davon ab, wie sich genau die Expansion des Universums über die Zeit entwickelt hat, aber dafür gibt es schon sehr gut passende Modelle, die durch Messungen der Entfernung von Galaxien über Supernovae kalibriert wurden.

    Was relativ ist, ist die Geschwindigkeit von Zeitabläufen und die Gleichzeitigkeit aus der Sicht von Beobachtern mit verschiedener Geschwindigkeit. Für uns, die wir uns in Ruhe betrachten, sind alle Zeiträume und Geschwindigkeiten wohldefiniert, auch wenn ein schnell bewegter Beobachter zu anderen Werten kommen würde.

  24. #24 DosenDieter
    16. Juni 2011

    Nicht vergessen darf man auch, dass wenn eine Masse beschleunigt wird, ja Energie aufgewendet werden muss und die beschleunigte Masse in der Beschleunigung auch schwerer wird und dadurch noch mehr Energie benötigt um wieder schneller zu werden.

    Ein denkender Körper könnte nie auf Lichtgeschwindigkeit beschleunigt werden, da unendlich Energie von Nöten wäre – daher können auch nur die masselosen Objekte Lichtgeschwindigkeit erreichen.

  25. #25 BuckRogers
    16. Juni 2011

    Das mit dem Zeitraffer war natürlich Mumpitz. Und denken kann ich als Photon natürlich auch nicht:)
    Aber ich habe mich ja wegen der Masselosigkeit dazu entschieden ein Photon zu sein, wobei ich ja trotzdem nicht wüsste, was JETZT da abgeht. Es dauert ja nochmal so lange bis ich endlich dort bin.
    Aber wo ich grad ein Photon bin…
    Ich frag mich z.B. auch immer in was für einer Dimension das Lichtteilchen lebt. Ich weiß, es lebt nicht. Es vergeht ja nicht einmal Zeit für das Photon. Zugleich befindet es sich an einem unbestimmten, unbestimmbarem Ort, bis es irgendwo auftrifft. Also den Raum, wie er für uns existiert, gibt es für das Photon auch nicht, solange es nicht wechselwirkt. Und dann die Sache mit den verschränkten Teilchen… Gehen meine Gedanken in die richtige Richtung? Ich meine, aus unserer Sicht aggiert das Licht im Raum, wird abgelenkt von Masse u.s.w, aber aus der Sicht des Lichts, existiert die Raumzeit doch gar nicht. Da ist das Licht nur Energie, ohne Raum und Zeit, einfach da. Überall zur gleichen Zeit. So kann ich mir zumindest auch vorstellen, dass erst durch das Abkühlen die Welt entstanden ist, wenn erst nur (unendlich hohe) Energie da war, gab es natürlich keine Raumzeit. Die gab es erst als es kalt genug für die ersten Teilchen war… oder?!? Und da frag ich mich dann, ob man die Zeitabläufe überhaupt mit unserer erlebten Zeit vergleichen kann. Wenn sie im Cern noch ‘ne millionstel Sekunde (ich weiß grad nicht die exakte Zahl) näher am Urknall sind, wie es so oft gesagt wird, dann frag ich mich immer was das für eine Relevanz hat. Eine millionstel Sekunde, pff, ok. Da kann aus Urknallsicht natürlich eine Menge passieren, aber wenn ein Außenstehender dass liest oder hört, denkt der natürlich, was soll denn der Quatsch. Ist ja auch schlüssig das man so mit der Zeit rechnet. Seit mir vor 23 Jahren der Vater meines schweizer Austauschschülers, Mitarbeiter im Cern, erzählte wie nahe sie dem Urknall sind geht mir das Thema nicht mehr aus dem Kopf:)
    Geh jetzt lieber ein Bier trinken;)

  26. #26 973
    17. Juni 2011

    Anschaulich ausgedrückt kann man zumindest vermuten dass die sichtbare Fülle des Raumes mit schwarzen Löchern früher nicht höher gewesen sein sollte als heute

  27. #27 SCHWAR_A
    17. Juni 2011

    @Florian Freistetter:
    Wie lang brauch eigentlich ein SL, um sich zu formen? Reicht die Zeit von 1 Mrd. Jahren?

  28. #28 Alderamin
    17. Juni 2011

    Zugleich befindet es sich an einem unbestimmten, unbestimmbarem Ort, bis es irgendwo auftrifft.

    Aber nur aus Sicht des Photons. Aus unserer Sicht ist es schon stets irgendwo unterwegs (wenn man den Ort ganz genau wissen will, dann wehrt es sich, aber das ist wieder eine andere Geschichte).

    Ich meine, aus unserer Sicht aggiert das Licht im Raum, wird abgelenkt von Masse u.s.w, aber aus der Sicht des Lichts, existiert die Raumzeit doch gar nicht.

    Das Licht breitet bewegt sich halt auf dem kürzesten Weg an sein Ziel. Wir hatten den Prof in der Physik-Vorlesung damals auch schon gelöchert, woher die “Austauschteilchen” (zu denen auch das Photon gehört: es vermittelt die elektromagnetische Kraft) beim Abflug “wissen”, wo ihr Ziel ist, also z.B. von einem Elektron zum Proton. Auf der anderen Seite ist ihre Jetztzeit ja quasi das gesamte Weltalter, sie sind also aus ihrer Sicht gleichzeitig an Start und Ziel.

    Gestern ist Brian Greenes Buch, ein stattlicher Wälzer von 600 Seiten, eingetroffen, mal schauen, was der dazu sagt. Eine Zusammenfassung von Florian gibt’s schon.

    Wenn sie im Cern noch ‘ne millionstel Sekunde (ich weiß grad nicht die exakte Zahl) näher am Urknall sind, wie es so oft gesagt wird, dann frag ich mich immer was das für eine Relevanz hat. Eine millionstel Sekunde, pff, ok. Da kann aus Urknallsicht natürlich eine Menge passieren, aber wenn ein Außenstehender dass liest oder hört, denkt der natürlich, was soll denn der Quatsch.

    Der LHC am CERN ist eine Art Mikroskop, das in die Teilchen hineinschauen kann. Je schärfer man schauen will, desto mehr Energie braucht man. Es geht beim LHC zwar auch um Vorgänge des Urknalls, aber im wesentlichen will man ausloten, wie die Materie im Innersten aufgebaut ist, woher die Masse kommt, woraus die dunkle Materie besteht, wie man alle 4 Grundkräfte unter einen Hut bekommt usw. Wenn man das weiß, ist man mit der gesicherten Physik natürlich wieder etwas näher am Urknall, den man aber niemals experimentell erreichen kann. Das alles ist hinreichend komplex, das kann man Außenstehenden oft schwer vermitteln (und noch schwerer, warum sie dafür Steuern zahlen). Aus dem Verständnis der Natur ergeben sich später aber meistens Anwendungen, von denen jeder profitiert. In heutigen Gigabyte-Festplatten werden z.B. Quanteneffekte verwendet, um die hohe Speicherdichte zu erreichen. Außerdem möchte der Mensch wissen, wie die Welt funktioniert, in der er lebt, deswegen gibt ja überhaupt die Wissenschaft.

  29. #29 Florian Freistetter
    17. Juni 2011

    @Schwar_A: “Reicht die Zeit von 1 Mrd. Jahren? “

    Offensichtlich 😉 Und die Entstehung von Himmelskörpern geht recht schnell. Sterne und Planeten brauchen dazu auch nicht mehr als ~ ne Million Jahre. Und ein SL ist ja auch nichts anderes als ein sehr schwerer Stern.

  30. #30 Alderamin
    17. Juni 2011

    @Florian

    Geht man eigentlich davon aus, dass das supermassive Schwarze Loch in einer Galaxie gleich aus dem Kollaps der Gaswolke entsteht, oder muss zuerst einmal eine Sternengeneration durchlaufen werden, die ein mehrere stellare Schwarze Löcher hervorbringt, und die dann verschmelzen?

    Dass die Größe des zentralen Bulge mit der Größe des Schwarzen Lochs korreliert, spricht möglicherweise dafür. Aber dass massive Sterne bei ihrer Entstehung das einfallende Gas wegblasen, sollte einen solchen Kollaps eigentlich verhindern, bis diese Sterne erloschen sind, was in der Größenordnung von 100 Millionen Jahren liegt.

  31. #31 Florian Freistetter
    17. Juni 2011

    @Alderamin: “Geht man eigentlich davon aus, dass das supermassive Schwarze Loch in einer Galaxie gleich aus dem Kollaps der Gaswolke entsteht, oder muss zuerst einmal eine Sternengeneration durchlaufen werden”

    Wie die supermassiven SLs entstehen weiß man noch nicht wirklich. Es kann beides möglich sein. U.a. deswegen macht man ja auch solche Untersuchungen.

  32. #32 JSM
    17. Juni 2011

    Könnte es nicht sogar noch viel mehr Schwarze Löcher geben? Und könnten sie nicht die fehlende Materie sein, die man sich z.Z. mit dunkler Materie erklärt, und damit die dunkle Materie überflüssig machen? Woher weiß man denn, dass die berechnete Materie wirklich (annähernd) stimmt, wenn man immer wieder auf neue SL trifft? Oder hat das Eine mit dem Anderen nichts zu tun?

  33. #33 Alderamin
    17. Juni 2011

    @JSM

    Könnte es nicht sogar noch viel mehr Schwarze Löcher geben?

    Theoretisch schon, man sieht ja nur die, die gerade Materie schlucken.

    Und könnten sie nicht die fehlende Materie sein, die man sich z.Z. mit dunkler Materie erklärt, und damit die dunkle Materie überflüssig machen? Woher weiß man denn, dass die berechnete Materie wirklich (annähernd) stimmt, wenn man immer wieder auf neue SL trifft?

    Nein, die dunkle Materie, die sich durch ihre Schwerkraft verrät, ist um den Faktor 4 größer, als die Menge der baryonischen (“gewöhnlichen”) Materie, die beim Urknall entstanden sein kann, also muss sie aus exotischer Materie bestehen, die nur per Schwerkraft wechselwirkt. Die bildet aber keine schwarzen Löcher, weil sie nicht “verklumpen” kann.

    Man kennt den Anteil an gravitativer Materie z.B. aus der kosmischen Hintergrundstrahlung, die etwas über die Dichte der Materie aussagt, sowie aus den Rotationskurven von Galaxien und der Gravitationslinsenwirkung von Galaxienhaufen.

    Gravitative Materie umfasst die dunkle und die baryonische Materie. Den Anteil der baryonischer Materie kann man ausrechnen, wenn man die Prozesse bei der Enstehung von Wasserstoff und Helium (Nukleosynthese) im frühen Universum durchrechnet. Siehe https://de.wikipedia.org/wiki/Primordiale_Nukleosynthese#Verbindung_zu_anderen_kosmologischen_Modellen.

  34. #34 WolfgangK
    17. Juni 2011

    Supermassive schwarze Löcher können nach Wikipedia als Obergrenze aufgrund großer Gravitationskräfte nur bis zu 10 Milliarden Sonnenmassen schwer sein. Was passiert aber, wenn sich zwei supermassive schwarzen Löcher mit jeweils ca. 10 Milliarden Sonnenmassen begegnen? Können die verschmelzen oder umkreisen die sich ausschliesslich?

  35. #35 Alderamin
    17. Juni 2011

    @Wolfgang K

    Was in dem Wiki-Artikel steht, bezieht sich auf das Wachstum über eine Akretionsscheibe. Ich denke mal, wenn zwei supermassive Schwarze Löcher mit zusammentreffen, z.B. nach einer Galaxienkollision, dann werden sie auch stets verschmelzen, egal wie groß sie sind, der im Wiki-Artikel beschriebene Prozess kann dann nicht greifen. Aber ich kenne die Originalarbeit nicht.

  36. #36 Niels
    17. Juni 2011

    @WolfgangK
    Na ja, aus dem Wiki-Artikel:

    Natarian und Treister[3] haben ein Modell entwickelt, das eine obere Massengrenze in der Größenordnung von 10 Milliarden Sonnenmassen vorhersagt.
    […]
    Den bisherigen Rekord stellt ein Schwarzes Loch von 18 Milliarden Sonnenmassen dar, welches im Quasar OJ 287 entdeckt wurde.

    Also können schwarze Löcher ganz offensichtlich größer als 10 Milliarden Sonnenmassen werden, nicht?

    Schwarze Löcher können durchaus verschmelzen, egal, wie groß sie sind.

  37. #37 Wurgl
    17. Juni 2011

    @Niels
    “Größenordnung von 10 Milliarden Sonnenmassen”

    18 Mrd ist in der Größenordnung von 10 Mrd. Da ist kein Widerspruch.

    Hier steht ein bissl was dazu: https://de.wikipedia.org/wiki/Größenordnung

  38. #38 Ralph Ulrich
    17. Juni 2011

    @Schwar_A: “Reicht die Zeit von 1 Mrd. Jahren? ”
    @Florian Freistetter: “Offensichtlich 😉 Und die Entstehung von Himmelskörpern geht recht schnell. Sterne und Planeten brauchen dazu auch nicht mehr als ~ ne Million Jahre. Und ein SL ist ja auch nichts anderes als ein sehr schwerer Stern.”

    Wenn man mal eine Computersimulation von Schwerkraftteilen im dreidimensionalen Raum gemacht hat, weiss man:
    Ein System von mehreren Teilen, in dem auch nur etwas Bewegung schon drin ist, fällt viel schwerer zusammen als eines, das mit einer Grundruhe anfängt. Weil sich bei einem vor-bewegenden System sofort Rotationen ergeben.

    Dagegen hatten die Urwolken nach dem Urknall erst einmal eine Grundruhe. Und sie zogen sich deswegen, ohne Rotation, einfach auf ihren Schwerpunkt zusammen. Dadurch nehme ich an, dass die ersten Sterne, die entstanden, viel größer waren im Durchschnitt, und viel schneller entstanden.

  39. #39 Niels
    17. Juni 2011

    @Wurgl
    Hm? Wo habe ich denn einen Widerspruch behauptet?
    WolfgangK schrieb:
    “Supermassive schwarze Löcher können nach Wikipedia als Obergrenze aufgrund großer Gravitationskräfte nur bis zu 10 Milliarden Sonnenmassen schwer sein.”

    Sowohl der Satz über das theoretische Modell als auch der Satz über die tatsächlichen Beobachtungsdaten in Wiki sagen etwas anderes aus als eine Obergrenze von “nur bis zu 10 Milliarden Sonnenmassen”, deswegen hab ich beide zitiert.
    Okay, vielleicht hätte ich Größenordnung auch noch fetten sollen.
    Allerdings hab ich nicht daran gedacht, dass das ohne Fettung missverständlich sein könnte.

  40. #40 Wurgl
    18. Juni 2011

    @Niels
    Hab ich halt falsch verstanden.

    Hier gabs vor ein paar Monaten ein Heftchen mit 2 Artikeln zu dem Thema
    https://www.wissenschaft.de/sixcms/detail.php?id=312964&template_id=9531&query_id=9532

    Und da ist auch ein Digramm drinnen (auf Seite 48, falls jemand das Heft zur Hand hat). Das Diagramm soll Leuchtkraft von Quasaren in Beziehung zur Masse der SL setzen. Bei 10 Mrd ändert sich die Punktdichte schlagartig und nur wenige Punkte sind darüber. Die 10 Mrd-Grenze passt also größenordnungsmäßig.

  41. #41 Rungi
    19. Juni 2011

    Wenn der Urknall 13 Mrd. Jahre her ist und nichts schneller als das Licht ist, wieso ist z.b. die Andromedagalaxie 250Mio. LIchtjahre entfernt? Diese Frage stellt sich mir beim Lesen der “Kurzen Geschichte der Zeit” von Hawkings.

  42. #42 Niels
    19. Juni 2011

    @Rungi
    Dein Problem kann ich bei diesem speziellen Beispiel zwar nicht nachvollziehen, aber schau mal hier rein:
    https://www.scienceblogs.de/hier-wohnen-drachen/2010/09/wie-gross-ist-das-beobachtbare-universum.php

  43. #43 Florian Freistetter
    19. Juni 2011

    @Rungi: Andromeda ist 2.5 Millionen Lichtjahre entfernt. 13 Milliarden sind 5200 mal mehr als 2.5 Millionen…

  44. #44 Rungi
    19. Juni 2011

    hmm 13Mrd Jahre = 13Mrd Lichtjahre?

  45. #45 Paul Zelmer
    19. Juni 2011

    @Alderamin
    Aber aus was besteht der Raum, dass er sich auseinander dehnt und sich nicht die Galaxien voneinander wegbewegen? Da müssen dann ja Raumteilchen sein? Bzw., hat dies etwas mit dem möglichen Big Rip zu tun?

    Ich glaube in der Quanten Schleifen Gravitation wird der Raum irgendwie so dargestellt, aber leider habe ich aus dem Bereich gar nichts verstanden 😀

  46. #46 rungi
    20. Juni 2011

    Ok, jetzt hab ich es endlich geschnallt.

  47. #47 Wurgl
    20. Juni 2011

    @Alderamin:

    Ich bin grad beim Schmökern meiner alten Hefte Spektrum der Wissenschaft und lese gerade im Heft Mai 2005 “Der Urknall — Mythos und Wahrheit”. Irgendwer hat das ins Netz gestellt, kann man hier nachlesen. https://homepage.univie.ac.at/Michael.Berger/lit/urknall.pdf

    Und da fällt mir folgendes auf und verursacht einen Knoten in meinen Gedanken. Jedenfalls wird dort von einem kosmischen Ereignishorizont gesprochen. Also eine Entfernung in der der Raum so schnell von uns wegexpandiert, dass dies schneller als die Lichtgeschwindigkeit ist und uns Licht aus dieser Gegend niemals erreichen wird.

    Gut, das ist auf in unserem “Zeiterleben” auch durchaus nachvollziehbar. Der Raum dehnt sich aus, die Zunahme an Raum zwischen uns und der Quelle ist so stark, dass in jeder Zeiteinheit mehr Raum hinzukommt als Licht in dieser Zeiteinheit zurücklegen kann.

    Gut. Akzeptiert.

    Nun aber dein Satz:
    “Auf der anderen Seite ist ihre Jetztzeit ja quasi das gesamte Weltalter, sie sind also aus ihrer Sicht gleichzeitig an Start und Ziel.”

    Für Photonen existiert quasi keine Zeit, wenn die gleichzeitig am Start und am Ziel sind, dann ist alles was sich zeitlich ändert — und damit eben auch die Expansion des Weltraums belanglos, den dazu müsste Zeit vergehen. Für ein Photon kann es (nach meinem Verständnis) diesen kosmischen Ereignishorizont nicht geben.

    Und da sehe ich einen Widerspruch zwischen unserem “Zeiterleben” und der Gleichzeitigkeit von Start und Ziel eines Photons.

    Wo ist jetzt der Wurm in meinen Gedanken?

  48. #48 Bjoern
    20. Juni 2011

    @Paul Zelmer: Wie kommst du von “der Raum dehnt sich aus” zu “es muss Raumteilchen geben”?

  49. #49 Bullet
    20. Juni 2011

    @Wurgl:

    Für ein Photon kann es (nach meinem Verständnis) diesen kosmischen Ereignishorizont nicht geben.

    Jo, das könnte gut sein. Jetzt der Trick: frag mal ein Photon, obs stimmt. 🙂

    Und da sehe ich einen Widerspruch zwischen unserem “Zeiterleben” und der Gleichzeitigkeit von Start und Ziel eines Photons.

    Wieso? Wir kennen so etwas wie “Zeit”, das Photon nicht. Das ist wie die Rennbahn aufm Sportplatz. Nur weil du 14 Sekunden brauchst, um dort hinten anzukommen, muß doch der Tartanstreifen nicht ebenso lange brauchen. Er ist bereits hier und da. Ein Photon würde sich selbst möglicherweise als vielfach geknickte und gekrümmte Linie beschreiben, dessen eines Ende sich am Emissionsort befindet und das andere Ende am Absorptionsort. Und das, was du seine zeitliche Dauer der Existenz nennst, wäre für es selbst nur seine Länge. (Übrigens ganz im Einvernehmen mit dem Konzept des Wortes “Raumzeit””, wie mir gerade auffällt.)

  50. #50 Ralph Ulrich
    20. Juni 2011

    @Bjoern, muss es nach der String Theorie nicht auch “Raumteilchen” geben?

    Aber, die Frage bleibt offen, ob man nicht “dunkle Energie” und “dunkle Materie” plus eine “in Wirklichkeit” stärkere Gravitation ähnlich einer MOND Theorie heraus kürzen kann:

    – Wenn Gravitation in Wirklichkeit nur Raumkrümmung ist,
    – Raumkrümmung ein Kontinuum ist,in den Voids total extrem,
    – bei gleichmäßigerer Verteilung der Materie Newtonsche Gravitationsstärken zeigt, die wir erleben.

    Wenn die Existenz von Materie einhergeht mit zusätzlichen, für uns unsichtbaren Dimensionen, die kurz sein mögen (Lisa Randall), aber Nachbarschaften provozieren (*), die über mittlere Strecken Raumkrümmungen hervorrufen, so dass MOND Phenomene erzeugt werden.

    (*) die zweidimensionale Welt eines Blatt Papiers kann man so krümmen, dass zwei entfernte Punkte sich berühren, obwohl diese “zusätzliche” Dimension der Krümmung minimal ist.

  51. #51 Wurgl
    20. Juni 2011

    @Bullet

    Ich hab mich nicht so ganz klar ausgedrückt oder den Begriff “kosmischen Ereignishorizont” anders, eventuell falsch verstanden.

    Mein Verständnis von “kosmischer Ereignishorizont” entspricht WIkipedia:
    https://de.wikipedia.org/wiki/Beobachtbares_Universum#Ereignishorizont
    ##
    gibt der Ereignishorizont an, wie weit ein Objekt heute maximal von uns entfernt sein darf, so dass uns sein Licht irgendwann in der Zukunft noch erreichen wird.
    ##

    Also eine Entfernung in der ein Photon uns niemals nie nicht erreichen kann.

    Aus der Sicht des Photons vergeht aber keine Zeit und wie Alderamin geschrieben hat, ist es “gleichzeitig” am Start und am Ziel.

    Der kosmische Ereignishorizont würde also eine Begrenzung der Reichweite des Photons bedeuten. Nur das passt wieder nicht so ganz zu anderen Texten wo die Reichweite eines Austauschteilchens von der Masse dieses Teilchens abhängt (via Unschärferelation) und dort wird bei masselosen Photonen von unendlicher Reichweite gesprochen.

  52. #52 Bjoern
    20. Juni 2011

    @Ralph Ulrich:

    muss es nach der String Theorie nicht auch “Raumteilchen” geben?

    Also, ich kenne die String-Theorie zwar nur oberflächlich, aber das wäre mir neu…

    Wenn Gravitation in Wirklichkeit nur Raumkrümmung ist,

    Äh, genau das sagt doch schon die ART?!?

    – Raumkrümmung ein Kontinuum ist,in den Voids total extrem,
    – bei gleichmäßigerer Verteilung der Materie Newtonsche Gravitationsstärken zeigt, die wir erleben.

    Den Teil verstehe ich nicht.

    Und: ist dir eigentlich klar, dass die derzeitige Theorie MIT Dunkler Energie und Dunkler Materie genau die beobachteten Voids, Filamente usw. vorhersagt? Wie kannst du also die Voids benutzen, um damit Dunkle Energie und Dunkle Materie weg zu argumentieren…?

    Die restlichen Absätze verstehe ich auch nicht so ganz. Zusätzliche, “aufgerollte” Dimensionen sind ein bekanntes Konzept der Stringtheorie – aber was soll das mit “Existenz der Materie” und “MOND-Phänomene erzeugen” zu tun haben? Und was meinst du mit “Nachbarschaft provozieren”?

  53. #53 Alderamin
    20. Juni 2011

    @Paul Zelmer: Wie kommst du von “der Raum dehnt sich aus” zu “es muss Raumteilchen geben”?

    @Paul Zelmer, Bjoern

    Was Paul mit den “Raumteilchen” meint, ist vielleicht die Plancksche Elementarlänge.

    Zunächst einmal gibt es im Raum eine Vakuumenergie, die den Raum aufbläht.

    Wie das in einem gequantelten Raum im einzelnen vor sich geht – ob sich mit geringer Wahrscheinlichkeit ab und zu ein Plancksche Elementarlänge verdoppelt, ein Raumelement spontan entsteht, oder wie auch immer – keine Ahnung.

    Ich bin ja nun auch erst dabei, die Konzepte halbwegs zu verstehen, hab’ Brian Greenes Buch mittlerweile gekauft, ist ein ziemlicher Wälzer von 600 Seiten, wird ein paar Tage dauern, die zu lesen. Vielleicht kann ich danach etwas kompetenter antworten.

  54. #54 Niels
    20. Juni 2011

    @Wurgl

    Für Photonen existiert quasi keine Zeit, wenn die gleichzeitig am Start und am Ziel sind, dann ist alles was sich zeitlich ändert — und damit eben auch die Expansion des Weltraums belanglos, den dazu müsste Zeit vergehen. Für ein Photon kann es (nach meinem Verständnis) diesen kosmischen Ereignishorizont nicht geben.
    Und da sehe ich einen Widerspruch zwischen unserem “Zeiterleben” und der Gleichzeitigkeit von Start und Ziel eines Photons.

    In der Kosmologie gibt es verschiedene Abstandsbegriffe. Hier sprechen wir bisher über die sogenannte “proper distance”. Man kann aber auch die “comoving distance” betrachten.
    https://en.wikipedia.org/wiki/Comoving_distance
    Diese Entfernung ändert sich bei Expansion des Universums nicht.
    So einen Entfernung, der sich nicht mit der Zeit ändert, hast du dir dich für das Photon gewünscht, oder?
    Partikelhorizont und Ereignishorizont sind nicht konstant, sondern ändern sich jeweils mit der Zeit.

    Den Partikelhorizont in “comoving distance” berechnet man mit der Formel https://upload.wikimedia.org/math/7/2/a/72ab06ecba2271c876f7d07a28f120ca.png .
    a (t) ist der sogenannte Skalenfaktor.
    Wenn man das von Null bis unendlich integriert, bekommt man einen endlichen Wert, nämlich etwa 65 Milliarden Lichtjahre. Der Partikelhorizont entwickelt sich also gegen einen Grenzwert.
    (In “proper distance” geht der Partikelhorizont dagegen selbstverständlich gegen unendlich. Der heutige Wert in “proper distance” ist 48 Milliarden Lichtjahre.)
    Aus diesem Grenzwert kann man also folgende Information ablesen: Wie weit darf ein Objekt heute von uns entfernt sein, damit Licht, dass dieses Objekt irgendwann einmal ausgestrahlt hat, uns jemals erreichen können wird?
    Das ist eben diese Entfernung von 65 Milliarden Lichtjahren.

    Licht kann also in mitbewegten Koordinaten (comoving distance) auch in unendlicher Zeit nur eine endliche Entfernung zurücklegen (da das Universums beschleunigt expandiert).
    Photonen haben also in “proper distance” eine unendliche Reichweite, in “comoving distance” allerdings eine endliche Reichweite. Irgendwann durchqueren sie nur noch Raum, den es sozusagen noch gar nicht gab, als sie losgeflogen sind. Sie verringern die “comoving distance” also nicht mehr.
    Deswegen können sie uns niemals erreichen.
    Hilft das, den Widerspruch auszulösen?

    Oder noch mal anders:
    Den Ereignishorizont berechnet man in “comoving distance” über https://upload.wikimedia.org/math/8/5/4/8546bfc665b1de29139455d6dfd8432a.png .
    Für t = 0 erhält man den Wert 65 Milliarden Lichtjahren, für t gegen unendlich geht der Ereignishorizont gegen Null.
    In “proper distance” geht der Ereignishorizont dagegen für t gegen unendlich gegen den endlichen Wert 16,2 Milliarden Lichtjahre.
    (Der heutige Wert in “proper distance” ist 15,6 Milliarden Lichtjahre.)
    Daraus, dass der Ereignishorizont in “physikalischen” (proper distance) Koordinaten einen Grenzwert hat bzw.in mitbewegten Koordinaten (comoving distance) gegen Null geht, kann man wieder etwas ablesen. Da sich Galaxien, die nicht gravitativ an unsere Galaxie gekoppelt sind, in “physikalischen” Koordinaten durch die Expansion immer weiter entfernen bzw. sie in mitbewegten Koordinaten immer den selben Abstand von uns haben, verschwinden irgendwann alle nicht gravitativ gebundenen Galaxien hinter den Ereignishorizont. Licht, das von Galaxien ausgeht, die sich jenseits des kosmischen Ereignishorizonts, befinden, kann uns niemals erreichen. Der Raum zwischen ihnen und uns expandiert zu schnell. Wir werden zwar solche Ereignisse sehen können, die in diesen Galaxien stattfanden, bevor sie den Horizont überquerten. Aber nachfolgende Ereignisse werden für alle Zeiten außerhalb unserer Sichtweite bleiben.
    Das heißt, irgendwann können wir außer der lokalen Galaxiengruppe keine anderen Objekte mehr sehen.

    Alle Klarheiten beseitigt? 😉

  55. #55 Ralph Ulrich
    20. Juni 2011

    @Bjoern sagte: “Und: ist dir eigentlich klar, dass die derzeitige Theorie MIT Dunkler Energie und Dunkler Materie genau die beobachteten Voids, Filamente usw. vorhersagt? Wie kannst du also die Voids benutzen, um damit Dunkle Energie und Dunkle Materie weg zu argumentieren…?”

    Ich will ja nicht die Beobachtungsdaten bestreiten, sondern das Paradigma “dunkle Energie”. Nach E=m*c*c bedeutet dies ja ein Äquivalent zu Materie und wird auch so dargestellt in den Tortendiagrammen: 75 Prozent.
    Ich glaube die “dunkle Energie” ist eine Folge der “non-fitting” Raumgeometrie in besonders leeren Räumen (Voids). Dieses “non-fitting” ist eine Idee von mir.

    Kontinuum war gemeint in Hinblick auf meine Frage viel weiter oben, ob Voids eine negative Krümmung hervorrufen können. Darauf antwortete Freistetter: Wo nichts ist , kann nichts gekrümmt sein.

    Nachdenken über diese Antwort, bringt mich zu dem Schluß, dass das Vorzeichen für meine Idee irrelevant ist. Wenn man die Raumkrümmung in Zahlen von zB 0 bis 100 festlegt, dann ist zB ein schwarzes Loch 100, aber die “fitting-Geometrie” 3 Sekunden nach dem Urknall vielleicht bei 3 Prozent.

    In leereren Räumen kann die Vakuum Energie neuen Raum bewirken, so wie diese Energie bei 100 Prozent an scharzen Löchern die Hawking Strahlung verursacht. Dann ist nur die Frage, was bewirkt diesen Effekt des “dünnen” Raumes überhaupt? Wie kann man es sich genau vorstellen?

    Und dazu brauche ich in meiner Vorstellung auch irgendwelche Raumteilchen in Planckgröße:
    Wenn Materie versteckte kurze Dimensionen hat, brauchen diese Dimensionen Nachbarn. Nachbarn sind für mich das Wesenselement von Dimensionen. Wenn sich Materie ansammelt, zieht sie diese Nachbarn mit sich mit, und diese ziehen andere Nachbarn mit (Raumteilchen). Dadurch entsteht der “dichtere” gekrümmte Raum, und andererseits entsteht in den Voids, aus denen diese Materie abgezogen wurde, eine Raumgeometrie, die dünner ist als die nach dem Urknall entstandene “fitting” Raumgeometrie von vielleicht 3 Prozent. Diese Zahl “3 Prozent” ist jetzt nur willkürlich als anschauliches Beispiel genannt, ich kann es als nicht Physiker nicht ausrechnen. Ausserdem müsste man ein guter Mathematiker mit Spezialgebiet Geometrie sein, um aus dieser Idee eine wirkliche “physikalische” Theorie zu machen.

    Irgendwo gab es mathematisch geometrische Annimationen zu sehen, die zeigen, wie ein vieldimensionaler Kubus projeziert auf unsere Geometrie aussehen würde: Im Innern wurde er immer verkruselter. So stelle ich mir Raumkrümmung an schweren Objekten vor.

  56. #56 Alderamin
    20. Juni 2011

    @Wurgl

    Man kann es sich vielleicht in einem Raum-Zeit-Diagramm klar machen. Nimm’ an, die x-Achse ist eine der drei Raumdimensionen, die y-Achse ist die Zeit, dann ist eine senkrechte Linie ein Objekt an einem festen Ort und eine geneigte Linie eine Bewegung von einem Ort zu anderen (zeichne eine waagerechte Linie zu dem Zeitpunkt, wo die Bewegung anfängt und eine weitere dort, wo sie aufhört; der Abstand der Linien auf der y-Achse ist die Dauer der Bewegung aus Sicht eines ruhenden Beobachters).

    Die Achsen seien so skaliert, dass Lichtgeschwindigkeit einem 45°-Winkel der Linie entspricht.

    Nun sollte man meinen, das Licht käme ja überall hin. Egal, wie weit mein Zielort entfernt ist, die zugehörige senkrechte Linie muss ja von der 45°-Linie der Photonenbewegung irgendwann mal geschnitten werden.

    Dies gilt aber nur in einem nicht expandierenden Raum. In einem expandierenden Raum laufen die x-Koordinaten nach oben auch auseinander, und zwar umso mehr, je weiter sie voneinander entfernt sind. Man stelle sich parallel und oberhalb der x-Achse mit den Ortsmarkierungen 1,2,3 etc. eine zweite x-Achse vor, auf der die Abstände der Ortsmarken verdoppelt sind. Und im gleichen Abstand darüber eine Achse, auf der sie verdreifacht sind, usw. Ein fester Ort entspricht dann über die Zeit einer Geraden, die gleiche Ortsmarken auf den einzelnen Achsen durchschneidet. Diese Geraden verlaufen mit zunehmendem Betrag von x immer flacher. Das ist gemessen in “proper distance”. Wenn nun das Licht seinen Weg als 45°-Linie startet, kann es einige der Linien fester Orte schneiden, aber irgendwann sind diese auch 45° oder mehr geneigt, d.h. das Licht kommt da nie hin, wenn es am Ursprung des Koordinatensystems losfliegt.

    Das entspricht der Situation einer Hubble-Expansion mit festem Hubble-Parameter. In dem Fall sind die Linien fester Orte Geraden, und wenn das Licht etwa vom Koordinatenursprung die Ortsmarke 10 erreichen kann, indem seine 45°-Linie die Linie dieser Ortsmarke irgendwann schneidet, so kann es das auch ausgehend von einem späteren Zeitpunkt (x-Koordinate 0, y-Koordinate t>0) stets wieder, aber der Weg wird länger, es dauert länger, die Marke von 0 aus zu erreichen.

    In einem Universum mit beschleunigter Expansion (wie wir es haben) sind die Linien fester Orte zusätzlich nach außen gebogen, d.h. der Abstand nimmt nicht linear zu, sondern exponentiell. D.h. aber, je später das Licht auf den Weg geht, desto weniger Ortsmarken kann es überhaupt noch erreichen. Sein Horizont wird immer kleiner. Lawrence Krauss hat in seinem Vortrag ausgerechnet, dass wir irgendwann nur noch die Milchstraße überblicken werden können, die Raumexpansion hat dann alle Galaxien hinter den dann gültigen Horizont weggetragen.

    So, und nun noch aus Sicht des Photon: Für das Photon ist die y-Achse eben keine Zeitachse, sondern eine Raumdimension und es sieht sich als 45°-Linie zwischen zwei Koordinaten in dem eben beschriebenen Diagramm. Auf diese Weise kann es eine unendlich lange Linie sein und trotzdem manche Ortskoordinaten nicht verbinden, weil die mehr als 45°-Neigung haben. Und je weiter oben die Lichtlinie losgeht, desto mehr Ortslinien haben diese stärkere Neigung, desto kleiner wird sein Horizont.

    Vielleicht mal auf einen Zettel malen, sollte dann eigentlich klar sein.

  57. #57 Bjoern
    20. Juni 2011

    @Ralph Ulrich:

    Ich will ja nicht die Beobachtungsdaten bestreiten, sondern das Paradigma “dunkle Energie”. Nach E=m*c*c bedeutet dies ja ein Äquivalent zu Materie …

    Genauer: zu Masse.

    Ich glaube die “dunkle Energie” ist eine Folge der “non-fitting” Raumgeometrie in besonders leeren Räumen (Voids). Dieses “non-fitting” ist eine Idee von mir.

    Und was heißt das?

    Wenn man die Raumkrümmung in Zahlen von zB 0 bis 100 festlegt, dann ist zB ein schwarzes Loch 100, aber die “fitting-Geometrie” 3 Sekunden nach dem Urknall vielleicht bei 3 Prozent.

    Es ist mir schleierhaft, wie du zu diesen Zahlen kommst. (und was “fitting-Geometrie” überhaupt heißen soll) Das fängt schon ganz einfach an: Da Zahlen von 0 bis 100 zu verwenden, legt nahe, dass es eine größtmögliche Krümmung gibt. (1) Warum sollte das so sein? (2) Warum sollte ein Schwarzes Loch diese größtmögliche Krümmung haben? (3) Wo genau am Schwarzen Loch soll diese größtmögliche Krümmung sein? Im Zentrum (Singularität)? Oder am Ereignishorizont? Oder im kompletten Inneren? Oder…?

    In leereren Räumen kann die Vakuum Energie neuen Raum bewirken,…

    Häh?

    …so wie diese Energie bei 100 Prozent an scharzen Löchern die Hawking Strahlung verursacht.

    Ich habe das Gefühl, du bringst hier zwei zwar ähnliche, aber doch grundverschiedene Konzepte durcheinander…

    Dann ist nur die Frage, was bewirkt diesen Effekt des “dünnen” Raumes überhaupt?

    Wieder: Häh? Was meinst du überhaupt mit “dünner Raum”?

    Wenn Materie versteckte kurze Dimensionen hat, brauchen diese Dimensionen Nachbarn. Nachbarn sind für mich das Wesenselement von Dimensionen.

    Häh? Was soll denn ein Nachbar einer Dimension sein?!?

    Wenn sich Materie ansammelt, zieht sie diese Nachbarn mit sich mit, und diese ziehen andere Nachbarn mit (Raumteilchen).

    Häh?!?

    Ausserdem müsste man ein guter Mathematiker mit Spezialgebiet Geometrie sein, um aus dieser Idee eine wirkliche “physikalische” Theorie zu machen.

    Verstehe ich dich richtig: du willst eine neue Theorie, um die Allgemeine Relativitätstheorie zu ersetzen?

    Irgendwo gab es mathematisch geometrische Annimationen zu sehen, die zeigen, wie ein vieldimensionaler Kubus projeziert auf unsere Geometrie aussehen würde: Im Innern wurde er immer verkruselter.

    Ich weiß nicht, wo du das her hast – wäre mir neu, dass so eine Projektion so aussieht… Bei Wikipedia sieht’s anders aus:
    https://de.wikipedia.org/wiki/Hyperkubus

    So stelle ich mir Raumkrümmung an schweren Objekten vor.

    Warum? Was hat das miteinander zu tun?

  58. #58 Ralph Ulrich
    20. Juni 2011

    @Bjoern,
    die englische Seite hat bessere Bilder als die deutsche:
    https://en.wikipedia.org/wiki/Hypercube
    “12-cube”

    Nachbarn, meine ich natürlich Punkte auf einer Dimension. Auch wenn die Dimension sehr kurz ist, wie Lisa Randall meint, muss sie Nachbarn ihrer Lokalitäten provozieren. Und diese Nachbarn können, wie das Beispiel eines gekrümmten Blattes zeigt, weit entfernt sein. Wo genau diese Nachbarn zu finden sind, müsste ein geometrisch ambitionierter Mathematiker darstellen.

    Die “dunkle Energie” wird oft in Publikationen mit der Vakuum Energie in Verbindung gebracht.

  59. #60 Niels
    20. Juni 2011

    @Ralph Ulrich
    Das Problem ist nur, dass die dunkle Energie leider mehr als hundert Größenordnungen (!!) kleiner ist als die Vakuum-Energie.
    https://en.wikipedia.org/wiki/Vacuum_catastrophe

  60. #61 Alderamin
    20. Juni 2011

    @Niels

    Aber hallo! Danke für den Link!!

  61. #62 Ralph Ulrich
    20. Juni 2011

    Kleiner ist viel besser für meine Theorie:
    Die Vakuum-Energie soll sich ja nur “aus Versehen”, wenn die Raum Geometrie nicht “fitting” ist, zu Beschleunigung/neuem Raum umsetzen.

  62. #63 Alderamin
    20. Juni 2011

    @Niels

    Dann habe ich allerdings einen Fehler gemacht mit der konstanten 45°-Linie, denn das Licht legt in dem Diagramm eine gebogene Linie zurück. Ich sollte es öfters mit Dieter Nuhr halten … 😉

    Das untere der drei Diagramme in https://www.physics.uq.edu.au/download/tamarad/astro/scienceimages/Spacetime_diagrams.pdf zeigt auch, dass mit zunehmender Zeit der Raumbereich in Comoving Distance, den wir überblicken können, immer kleiner wird. Der weiße Bereich wird zunehmend schmaler und Licht jenseits des Ereignis-Horizonts erreicht uns nicht mehr, und der wird immer kleiner.

  63. #64 Alderamin
    20. Juni 2011

    @Florian

    @Alderamin: “Geht man eigentlich davon aus, dass das supermassive Schwarze Loch in einer Galaxie gleich aus dem Kollaps der Gaswolke entsteht, oder muss zuerst einmal eine Sternengeneration durchlaufen werden”

    Wie die supermassiven SLs entstehen weiß man noch nicht wirklich. Es kann beides möglich sein. U.a. deswegen macht man ja auch solche Untersuchungen.

    Wie ich soeben gelesen habe, hat man früher geglaubt, es müsse zuerst ein Black Hole aus einem Stern als Keim für das supermassive SL entstehen. Seit 2006 gebe es jedoch ein Paper, das annimmt, die supermassiven SLs entstünden sofort wie die Fixsterne in einem Schritt. So, wie es hier dargestellt ist, klingt das so, als sei das die aktuelle Auffassung.

  64. #65 Bjoern
    20. Juni 2011

    @Ralph Ulrich:

    die englische Seite hat bessere Bilder als die deutsche:

    Ah, danke. Und? Was hat das jetzt mit Raumkrümmung um schwere Objekte zu tun?!?

    Und warum hast du eigentlich gut 2/3 meiner Fragen ignoriert?

    Nachbarn, meine ich natürlich Punkte auf einer Dimension.

    Und was soll “Punkte auf einer Dimension” wieder heißen?

    …muss sie Nachbarn ihrer Lokalitäten provozieren.

    Und was soll das wieder heißen?

    Und diese Nachbarn können, wie das Beispiel eines gekrümmten Blattes zeigt, weit entfernt sein.

    Das widerspricht aber irgendwie der Bedeutung des Wortes “Nachbarn”… wie wär’s mit einem passenderen Begriff?

    (Niels:)Das Problem ist nur, dass die dunkle Energie leider mehr als hundert Größenordnungen (!!) kleiner ist als die Vakuum-Energie.
    (Ralph Ulrich:) Kleiner ist viel besser für meine Theorie:
    Die Vakuum-Energie soll sich ja nur “aus Versehen”, wenn die Raum Geometrie nicht “fitting” ist, zu Beschleunigung/neuem Raum umsetzen.

    Und wieder: was soll das nun wieder heißen? Inwiefern hilft es dir da, dass die dunkle Energie um mehr als 100 Größenordnungen kleiner ist als die Vakuum-Energie? Fangen wir doch mal ganz langsam an: weißt du überhaupt, was “100 Größenordnungen” überhaupt heißt…?

  65. #66 Niels
    20. Juni 2011

    @Alderamin
    Hast du beachtet, dass die Zeitachse beim untersten Bild in “conformal time” ist?
    Das ist ein für mich überhaupt nicht intuitiv zugängliches Zeitmaß.

    Übrigens hab ich die die Sache mit Ereignishorizont, Partikelhorizont usw. durch diese beiden Paper verstanden.
    https://arxiv.org/abs/astro-ph/0310808
    https://arxiv.org/abs/astro-ph/0402278v1
    (Das zweite Paper enthält das erste Paper größtenteils, in einer etwas anderer Formulierung und mit Diagrammen in Farbe.)

    Du bist Physiker, oder? Ich fand das ziemlich einfach zu verstehen, obwohl ich mit Kosmologie eigentlich nix zu tun habe.
    Da kommen auch die beiden oben verlinkten Bildchen her.

    @Wurgl
    Auf dem ersten Paper beruht übrigens der von dir verlinkte populärwissenschaftliche Artikel “Der Urknall – Mythos und Wahrheit”. Ist auch vom gleichen Autor.
    (Das ist übrigens der beste populärwissenschaftliche Artikel zur Kosmologie, den ich kenne.)
    Während der populärwissenschaftliche Artikel notgedrungen nur beschreibt, erklärt die wissenschaftliche Veröffentlichung das Ganze tiefergehend.

    Du kannst da ja mal reinschauen, für einen Nichtphysiker aber vielleicht ein bisschen zu schwer.

  66. #67 Ralph Ulrich
    20. Juni 2011

    @Bjoern, wenn ich als Nicht-Mathematiker mein einfaches Blatt Beispiel bringe, und Du es nicht verstehen willst, ist es rein technisch hier schwer auf alles zu antworten. Nun mathematischer formuliert:

    Bringe zwei Ebenen aneinander und du hast einen dreidimensionalen Raum, dessen dritte Dimension zwei Punkte groß ist.

    Der Knick im Blatt Papier Beispiel symbolisiert den Umweg, den ein ZweiDim Mensch gehen müsste um seinen direkten Nachbarn zu erreichen, dem ein DreiDim Mensch direkt die Hand geben kann. Mein Gefühl für Dimensionen (also Nicht wissenschaftlich) sagt mir nun:

    Ließen sich zusätzliche Dimensionen nachweisen, dann enden sie nicht im Leeren, seien sie auch noch so kurz.

    Sind hundert Größenordnungen = x**100 ? Wundert mich schon dieser Größenunterschied, weil es doch sagt, dass die “dunkle Energie” ein Mini-Mini-Mini-etc Teil der Vakuumenergie ist, aber sie soll 70 Prozent der Gesamtenergie der Universums ausmachen und sie soll damit zwanzigfach größer als die Baryonenmasse sein? Oder ist hier jetzt nur der Rechenfehler gemeint, der passiert, wenn man mit quantenmechanischen Formeln die Vakuumenergie ausrechnet und dabei auf eine Sonnenmasse pro Kubikzentimeter kommt 🙂

  67. #68 Alderamin
    20. Juni 2011

    @Niels

    Hast du beachtet, dass die Zeitachse beim untersten Bild in “conformal time” ist?
    Das ist ein für mich überhaupt nicht intuitiv zugängliches Zeitmaß.

    Schon, im Text steht, dass die unendliche Zeit hier auf eine endliche Zeit abgebildet wird. Hier sind die meisten Linien gerade, was das Bild übersichtlicher macht. Viel weiter habe ich in dem Moment des Posts nicht gedacht, hab’ dies aber jetzt nachgeholt:

    Wenn das Universum so, wie es jetzt ist, eingefroren würde und die aktuellen (Comoving) Enfernungen unverändert blieben, dann wäre die Conformal Time die Zeit, die ein Lichtstrahl braucht, um eine bestimmte Strecke in der eingefrorenen Entfernung zurückzulegen (also 1 Jahr Conformal Time = Zeit, in der ein Lichtstrahl 1 Lichtjahr Comoving Distance zurücklegt). So also ob das Weltall gar nicht expandierte. Sie ist die zeitliche Entsprechung der Comoving Distance, deswegen werden die Linien der Lichtgeschwindigkeit wieder gerade mit 45° Neigung, wie in einem nicht expandierenden Universum.

    Licht, das uns jetzt (nach 13,7 Milliarden Jahren Eigenzeit) erreicht, wurde von Objekten ausgesendet , die in Comoving Distance mittlerweile 46 Milliarden Lichtjahre entfernt sind. Deswegen ist uns eine Conformal Time von 46 Milliarden Jahren zugeordnet. Licht, das von Objekten ausgeht, die derzeit 61 Milliarden Lichtjahre weit weg sind, kann uns erst nach unendlicher Eigenzeit erreichen (d.h. eben gerade nicht mehr), deswegen ist dem Zeitpunkt unendlich eine Conformal Time von 61 Milliarden Jahren zugeordnet. Dies definiert unseren Horizont, wir werden niemals sehen, was dahinter ist.

    Nein, die Definition der Conformal Time ist nicht sehr intuitiv, aber macht schön übersichtlich, wie weit man früher, jetzt und jemals gucken konnte/kann/können wird.

    Du bist Physiker, oder? Ich fand das ziemlich einfach zu verstehen, obwohl ich mit Kosmologie eigentlich nix zu tun habe.

    Leider nicht, promovierter Informatiker mit Nebenfach Physik und mit 3 belegten Vorlesungen in Astronomie. Ist aber schon ein paar Jahre her, damals ging man noch von einer sich verlangsamenden Expansion aus. Dass die Geometrie des expandierenden Universums so kompliziert ist, war mir in der Tat neu.

  68. #69 Wurgl
    20. Juni 2011

    @Niels
    @Alderamin

    War beschäftigt. Muss zwischendurch auch mal Geld verdienen …

    Ich hab kein Problem mit der Betrachtung als “ruhender” Beobachter. Die Diagramme hab ich schon vor langer Zeit kapiert, die sind einleuchtend. Ich hab nur ein Problem mit der Betrachtung aus der Sicht des Photons. So ganz kapier ich noch nicht warum die y-Achse (also meine Zeit) plötzlich eine Raumachse werden soll. Wie ist das bei fast-lichtschnellen Teilchen, ist es dann Raumachse/siebenzehntel? Da wären Links zum Nachlesen hilfreich.

  69. #70 Bjoern
    20. Juni 2011

    @Ralph Ulrich:

    Bringe zwei Ebenen aneinander und du hast einen dreidimensionalen Raum, dessen dritte Dimension zwei Punkte groß ist.

    “Zwei Punkte” ist keine Größe. Ein Punkt hat die “Größe” Null, zwei Punkte zusammen haben also auch die Länge Null. (2*0 = 0) Schon das Konzept, Ebenen “aneinander zu bringen” ist nicht besonders sinnvoll – was soll das heißen? Dass die Ebenen keinen Abstand mehr zueinander haben? Dann sind sie aber identisch und liegen nicht “nebeneinander”.

    Dein Denkfehler hier scheint zu sein, dass du den Ebenen von vornherein schon eine Dicke größer als Null zuordnest – aber wo soll diese denn herkommen? Die Analogie mit dem Blatt Papier bricht hier zusammen – ein Blatt Papier ist halt nicht wirklich zweidimensional, sondern immer noch dreidimensional, auch wenn man die Dicke kaum wahrnimmt…

    Ließen sich zusätzliche Dimensionen nachweisen, dann enden sie nicht im Leeren, seien sie auch noch so kurz.

    Nach derzeitigem Verständnis (String-Theorie) enden die zusätzlichen Dimensionen ja überhaupt nicht, sondern sind “aufgerollt”.

    Sind hundert Größenordnungen = x**100 ?

    Was meinst damit? x hoch 100? Wenn ja: was ist x? (die richtige Antwort wäre übrigens: der Unterschied ist ein Faktor 10 hoch 100)

  70. #71 Niels
    20. Juni 2011

    @Alderamin
    Jo, das mit der “conformal time” hast du komplett richtig verstanden.
    Respekt, um diese Diagramme zu durchschauen hab ich damals ne Weile länger gebraucht.

    @Wurgl

    Ich hab nur ein Problem mit der Betrachtung aus der Sicht des Photons.

    Vielleicht ist das ja der Knackpunkt. Bleiben wir der Einfachheit halber mal bei der SRT.
    In welchem Inertialsystem befindet sich der Beobachter? Wie sieht eine Lorentz-Transfo in dieses Bezugssystem aus?
    Da ein Photon kein sinnvolles Ruhesystem besitzt, kann man auch seine Eigenzeit nicht definieren.
    Auch die Lorentz-Trafo wird singulär.
    Wenn man aber keinen sinnvollen Beobachter angeben kann, kann man vielleicht auch nicht sinnvoll darüber sprechen, oder?

    So ganz kapier ich noch nicht warum die y-Achse (also meine Zeit) plötzlich eine Raumachse werden soll. Wie ist das bei fast-lichtschnellen Teilchen, ist es dann Raumachse/siebenzehntel?

    Ich hab keine Ahnung, was hier gemeint ist. Kannst du das mal genauer ausformulieren? Vielleicht möglichst mit irgend einer Quelle?
    Warum wird die Zeitachse zur Raumachse? Warum “Raumachse/siebenzehntel?”? Gehts irgendwie um Längenkontraktion oder wie?

  71. #72 Wurgl
    20. Juni 2011

    @Niels:

    Aus dem Text von Alderamin:
    “Nimm’ an, die x-Achse ist eine der drei Raumdimensionen, die y-Achse ist die Zeit, …”

    Hier ist also auf y die Zeit eingetragen.

    und dann fast ganz unten weiter:
    “So, und nun noch aus Sicht des Photon: Für das Photon ist die y-Achse eben keine Zeitachse, sondern eine Raumdimension …”

    Das meinte ich mit der y-Achse und der Zeit bzw. dem Raum.

    Und wenn ich jetzt mal sage, dass ein ruhendes Objekt diese y-Achse als Zeit interpretiert/betrachtet und ein lichtschnelles als Raum, was ist dann bei einem fast lichtschnellen? Bei einem stetigen Übergang ist bei fast-Lichtgeschwindigkeit diese y-Achse eben teilweise Raum und teilweise Zeit.

    Und damit hab ich ein Gedankenproblem.

  72. #73 Ralph Ulrich
    20. Juni 2011

    Bjoern: “”Zwei Punkte” ist keine Größe. Ein Punkt hat die “Größe” Null, zwei Punkte zusammen haben also auch die Länge Null.”

    Die Länge zwischen zwei Punkten, die sich per Definition in direkter Nachbarschaft befinden: Ich wette, ein Mathematiker kann beweisen, dass sie größer Null sein muss. Im physikalischen Zusammenhang würde ich von der Planckzahl 10**-36 ausgehen um eine sinnvolle Diskussion zu führen.

    Zusätzliche Dimensionen in der Stringtheorie haben in ihrem aufgerollten Zustand wohl nur einen rechnerischen Sinn? Aber es gibt auch Forschung, die nach zusätzlichen Kurzdimensionen sucht, die eben nicht aufgerollt sind. Und dann hätten wir eben nicht-lokale Effekte in unserer dreidimensionalen Welt, wie dir das geknickte Blatt Papier als bildliches Beispiel vor Augen führen sollte in all seiner Begrenztheit der Metaphorik.

    … wären wir Vögel, hätten wir vermutlich eine Drei als Basis für Größenordnungen, da erlauben unsere zehn Finger sparsamere Potenzen …

  73. #74 Alderamin
    20. Juni 2011

    @Wurgl

    Na, Dein Punkt war doch, wenn das Photon aus seiner Sicht keine Zeit kennt, kommt es überall hin, weil die Ausdehnung des Weltalls keine Rolle spielt. Man kann die von Niels verlinkten Bilder auch einfach als Karten interpretieren, die von außen gesehen ein statisches Universum zeigen, in dem das Photon (gerade beim 3. Bild mit der Conformal Time) ein 45°-Strich auf der Karte ist. In Greenes Buch ist von einem eingefrorenen Universum die Rede, der Zeitfluss ist nur scheinbar für diejenigen, die in ihm leben. Und sich mit weniger als Lichtgeschwindigkeit bewegen.

    Ist vielleicht etwas ungeschickt ausgedrückt, dass das Photon die Zeitachse als Raumachse betrachten könnte. Alle 3 Raumdimensionen und die Zeit bilden halt ein 4-dimensionales Spielfeld, in dem das Photon ein Strich mit 45° Winkel zur Zeitachse ist. Das Photon könnte sagen: ich bin am Start und am Ziel zugleich und ich bin eine Gerade, die zwei Punkte auf dem Spielfeld mit 45° Neigung zur y-Achse verbindet. Das wäre eine zeitlose Beschreibung.

    Und sie verbietet es (siehe Conformal-Time-Diagramm), dass das Photon trotz unendlicher Zeit von jedem Ort an jeden anderen gelangen kann. Bei 65 Milliarden Jahren Conformal Time ist die Linie oben spätestens zu Ende, also kommt sie bei 45° Neigung auch nicht weiter als 65 Milliarden Lichtjahre vom Startpunkt weg, wenn sie gleich unten beim Urknall beginnt.

  74. #75 Bjoern
    20. Juni 2011

    @Ralph Ulrich:

    Die Länge zwischen zwei Punkten, die sich per Definition in direkter Nachbarschaft befinden: Ich wette, ein Mathematiker kann beweisen, dass sie größer Null sein muss.

    Nein, jeder Mathematiker wird dir sagen, dass “zwei Punkte in direkter Nachbarschaft” schlichtweg keinen Sinn ergibt (in einem Kontinuum; wenn der Raum quantisiert sein sollte, sieht’s eventuell anders aus – siehe unten). Ich bin zwar “nur” Physiker, aber ich habe wirklich genügend Mathematik-Vorlesungen gehört, um praktisch jede beliebige Summe auf diese Aussage zu verwetten.

    Im physikalischen Zusammenhang würde ich von der Planckzahl 10**-36 ausgehen um eine sinnvolle Diskussion zu führen.

    Dir ist aber schon klar, dass es bisher nur eine Hypothese ist, dass der Raum tatsächlich quantisiert ist?

    Zusätzliche Dimensionen in der Stringtheorie haben in ihrem aufgerollten Zustand wohl nur einen rechnerischen Sinn?

    Mir ist nicht klar, was “rechnerischer Sinn” hier heißen soll…

    Aber es gibt auch Forschung, die nach zusätzlichen Kurzdimensionen sucht, die eben nicht aufgerollt sind.

    Könnte durchaus sein – in dem Bereich bin ich alles andere als ein Experte. Wo hast du das denn her?

    Und übrigens ist mir immer noch nicht klar, was das ganze denn nun mit Dunkler Energie / Materie zu tun haben soll… Z. B. könntest du mal erklären, was du denn mit “dichterem/dünneren Raum” meinst.

  75. #76 Ralph Ulrich
    21. Juni 2011

    @Bjoern, die Idee ist, der Raum in den Voids ist anders als wir ihn kennen. Dort kann es dann vermehrt vorkommen, dass die Vakuumenergie ähnlich verrückt spielt, wie an schwarzen Löchern, wo sie Hawking Strahlung verursacht. Dort aber neuen Raum erzeugt, einen Effekt erzeugt, den wir als “dunkle Energie” bezeichnen.

    Was ist offensichtlich anders in den Voids: Es gibt keine Materie. Wie wirkt Materie auf den Raum: Sie krümmt ihn. Offensichtlich braucht der Raum eine gewisse Krümmung um ohne Fehler normal zu funktionieren. Das nenne ich “geometrisches fitting” des normalen Raumes, wie wir ihn kennen.

    Ich habe zwei Möglichkeiten der Erklärung dieses “non-fitting” in den Voids im Kopf, beide brauchen Raumteilchen:
    1. Materie entzog den Voids Raumteilchen, als sie abzog.
    2. Materie bedient in Fernwirkung (geknicktes Blatt!) über zusätzliche Dimensionen Kanten der Raumteilchen, die in den Voids als Leerstellen offen bleiben. Also eine Waben- oder Würfelstruktur, die in den Voids unausgeglichen bleibt und zu stark “wackelt”.

    Beide mögliche Erklärungen beinhalten die Notwendigkeit, dass Materie in ihrem Volumen nicht vollständig dreidimensional erklärbar ist. Und ich glaube auch, dass letztendlich “zufällige” Zerfallsprozesse durch “wackelnde” Geometrie erklärt werden können. Denn alles was schwerer als Wasserstoff ist, ist auch “non-fitting”. Es werden Ausgleichsschwingungen auftreten, die durch “Zufall” sich aufschaukeln können, dass eine Kraft entsteht, die das Atom auseinander bricht.

    Wenn man die Zeichnungen von kompliziertesten stringtheoretischen Teilchen in mehreren Dimensionen gesehen hat, weiss man, wie schwer es werden wird davon rechenbare Modelle zu machen. Aber ich bin mir sicher, dies wird gemacht werden.

  76. #77 Ralph Ulrich
    21. Juni 2011

    Schildkrötenparadoxon als Herleitung, dass zwei Punkte nebeneinander doch auch mathematisch etwas hergeben müssten: Wenn
    – zwei Punkte nebeneinander eine Strecke Null ergeben
    – dann drei Punkte auch
    – dann unendlich viele Punkte auch
    – eine Gerade ist durch unendlich viele Punkte direkt nebeneinander definiert.
    Da aber eine Gerade unendliche Länge hat …

    In der Antike gab es schon mal ein ähnliches Gedankenexperiment mit einem Wettlauf mit einer Schildkröte …

  77. #78 Alderamin
    21. Juni 2011

    @Ralph Ulrich

    Schildkrötenparadoxon als Herleitung, dass zwei Punkte nebeneinander doch auch mathematisch etwas hergeben müssten: Wenn
    – zwei Punkte nebeneinander eine Strecke Null ergeben
    – dann drei Punkte auch
    – dann unendlich viele Punkte auch
    – eine Gerade ist durch unendlich viele Punkte direkt nebeneinander definiert.
    Da aber eine Gerade unendliche Länge hat …

    So kann man mit Unendlichkeiten nicht umgehen. Deine Punkte sind abzählbar wie die Brüche (Rationale Zahlen). Angenommen, Du hättest zwei Punkte, die Deiner Meinung nach direkt benachbart seien. Sagen wir x und y. Dann setze ich mit Leichtigkeit noch einen dazwischen: z=(x+y)/2. Und noch zwei: (x+z)/2 und (z+y)/2. Für diese Punkte gilt dasselbe wie für x und y: ich kann immer noch weitere Punkte dazwischen setzen, und das hört nie auf. Zwischen Deinen zwei benachbarten Punkte kann ich also unendlich viele weitere finden, und bin dann immer noch in einer abzählbaren Menge (ich kann die erzeugten Punkte in der Reihenfolge abzählen, in der ich sie gebildet habe). Ohne dass sich Deine beiden Ausgangpunkte x+y voneinander entfernt hätten. Auf die Weise gewinnst Du keinen Raum. Es sei denn, der wäre gequantelt und ließe sich irgendwann nicht weiter aufteilen.

    Die Schildkrötengeschichte haben wir im ersten Semester Mathematik mal an einer unendlichen Reihe ausgerechnet. Der Witz ist, dass Hase und Schildkröte in jedem betrachteten Zeitintervall ein Stückchen Weg (und nicht nur einen Punkt) zurücklegen, nur werden die betrachteten Strecken- und Zeitintervalle immer kleiner. Wenn man die Strecken oder Zeiten aber als unendliche Summe addiert, kann man ausrechnen, welchem Wert die Summe entgegenstrebt, der ist nämlich endlich (beschränkt). Da hat der Hase die Schildkröte dann eingeholt.

  78. #79 Alderamin
    21. Juni 2011

    @Bjoern, die Idee ist, der Raum in den Voids ist anders als wir ihn kennen. Dort kann es dann vermehrt vorkommen, dass die Vakuumenergie ähnlich verrückt spielt, wie an schwarzen Löchern, wo sie Hawking Strahlung verursacht. Dort aber neuen Raum erzeugt, einen Effekt erzeugt, den wir als “dunkle Energie” bezeichnen.

    Die Voids entstehen in Computersimulationen (wie oben im Artikel zu sehen) auch ohne die Annahme, dass der Raum nur in den Voids expandiert, indem die Materie sich mit der dunklen Materie zu fadenartigen Strukturen zusammenzieht und die Voids zurücklässt. Die neuesten Messungen deuten daraufhin, dass die dunkle Energie die Eigenschaft einer kosmologischen Konstante hat, d.h. der Raum dehnt sich überall gleich stark aus, die Ausdehnung ist eine Eigenschaft des Vakuums selbst. Sie hat die Eigenschaft einer Vakuumenergie, lässt sich aber von der Quantenmechanik nicht ableiten. Das Ergebnis ist um 120 Größenordnungen zu groß.

  79. #80 Ralph Ulrich
    21. Juni 2011

    @Alderamin, meine Frage wäre: Hat man schon Beobachtungsdaten, die zeigen ob die “dunkle Energie” sich unterscheidet in ihren Effekten
    – im interstellaren Raum der Galaxien
    – im intergalaktischen Raum der Galaxienhaufen
    – in den Voids

    Ich hatte kürzlich gelesen, dass eine Hubblerate von 32 in Galaxienhaufen gefunden wurde. Wenn man da dann die gravitativen Gebundenheiten herausrechnet, was kommen für die Effekte der “dunklen Energie” dabei heraus?

  80. #81 Alderamin
    21. Juni 2011

    Mit “wie oben im Artikel zu sehen” bezog ich mich auf die Simulation in diesem Artikel.

  81. #82 Alderamin
    21. Juni 2011

    @Alderamin, meine Frage wäre: Hat man schon Beobachtungsdaten, die zeigen ob die “dunkle Energie” sich unterscheidet in ihren Effekten
    – im interstellaren Raum der Galaxien
    – im intergalaktischen Raum der Galaxienhaufen
    – in den Voids

    Man kann ja nur die Expansion der Galaxien messen, aber man hat wohl keine Abhängigkeit von der Materiedichte gefunden. Ich kann dazu jetzt nichts verlinken, aber in der Juliausgabe von Sky & Telescope stand wieder drin, dass die dunkle Energie eine kosmologische Konstante sei, d.h. sie ist überall gleich.

    Man hatte als alternative Theorie die Möglichkeit einer 5. Grundkraft (Quintessenz) erwogen, die dann aber von der Materiedichte abhängig sein müsste (wie auch die Gravitation). Das scheinen die Messungen nicht zu bestätigen. Von der alternativen Theorie stand aber nichts in dem Artikel.

    Ich hatte kürzlich gelesen, dass eine Hubblerate von 32 in Galaxienhaufen gefunden wurde. Wenn man da dann die gravitativen Gebundenheiten herausrechnet, was kommen für die Effekte der “dunklen Energie” dabei heraus?

    Ich glaube, darauf bezog sich der S&T-Artikel. Der Hubble-Parameter kam ein wenig höher heraus, als aus den Messungen der Hintergrundstrahlung, so um die 73-74 km/s/Mpc, wenn ich mich recht entsinne. Über die dunkle Energie stand eben das oben gesagte drinnen.

  82. #83 Bjoern
    21. Juni 2011

    @Ralph Ulrich:

    die Idee ist, der Raum in den Voids ist anders als wir ihn kennen.

    Erstens einmal (wie übrigens ich schon gestern und Aldemarin ebenfalls schon sagte): die Entstehung der Voids wird bereits durch die Dunkle Materie / Energie etc. erklärt. Warum ein neues Modell? Zweitens wiederhole ich meine Frage gestern: verstehe ich dich richtig? Du willst also eine Alternative zur Allgemeinen Relativitätstheorie entwickeln…?

    Dort kann es dann vermehrt vorkommen, dass die Vakuumenergie ähnlich verrückt spielt, wie an schwarzen Löchern, wo sie Hawking Strahlung verursacht.

    Erstens einmal wurde dir schon erklärt, dass der Zusammenhang zwischen Vakuumenergie und Hawking-Strahlung alles andere als so einfach ist, wie du ihn hier hinstellst. Zweitens: wieso “verrückt spielt”?!?

    Dort aber neuen Raum erzeugt, einen Effekt erzeugt, den wir als “dunkle Energie” bezeichnen.

    Was hat das denn mit dunkler Energie zu tun?!?

    Offensichtlich braucht der Raum eine gewisse Krümmung um ohne Fehler normal zu funktionieren.

    ?????

    Ich habe zwei Möglichkeiten der Erklärung dieses “non-fitting” in den Voids im Kopf, beide brauchen Raumteilchen:
    1. Materie entzog den Voids Raumteilchen, als sie abzog.
    2. Materie bedient in Fernwirkung (geknicktes Blatt!) über zusätzliche Dimensionen Kanten der Raumteilchen, die in den Voids als Leerstellen offen bleiben. Also eine Waben- oder Würfelstruktur, die in den Voids unausgeglichen bleibt und zu stark “wackelt”. Beide mögliche Erklärungen beinhalten die Notwendigkeit, dass Materie in ihrem Volumen nicht vollständig dreidimensional erklärbar ist.

    ?????

    Entschuldigung, aber das ist schlicht und einfach wirr und nicht nachvollziehbar. Es ist weder erkennbar, wozu diese Erklärung gut sein soll, noch wie sie funktionieren soll.

    Und ich glaube auch, dass letztendlich “zufällige” Zerfallsprozesse durch “wackelnde” Geometrie erklärt werden können. Denn alles was schwerer als Wasserstoff ist, ist auch “non-fitting”. Es werden Ausgleichsschwingungen auftreten, die durch “Zufall” sich aufschaukeln können, dass eine Kraft entsteht, die das Atom auseinander bricht.

    (1) Du meinst wohl “Atomkern”, nicht “Atom” – oder? (2) Wieso sollte alles schwerer als Wasserstoff “non-fitting” sein? (3) Warum gibt es mehrere hunderte stabile Isotope?

    Wenn man die Zeichnungen von kompliziertesten stringtheoretischen Teilchen in mehreren Dimensionen gesehen hat, weiss man, wie schwer es werden wird davon rechenbare Modelle zu machen. Aber ich bin mir sicher, dies wird gemacht werden.

    Dir ist aber schon klar, dass es auf der Welt hunderte von Leuten mit alternativen Theorien gibt, die alle sagen “meine Mathekenntnisse sind nicht gut genug, rechne das mal für mich durch!” – oder?

    Wenn
    – zwei Punkte nebeneinander eine Strecke Null ergeben
    – dann drei Punkte auch
    – dann unendlich viele Punkte auch

    Der letzte Schritt ist falsch. Ein Argument, dass für eine beliebige endliche Zahl gilt, ist nicht automatisch im Unendlichen auch richtig. (und es gibt -zig Beispiele aus der Mathematik, die das zeigen)

    Außerdem gibt es sogar verschiedene Arten von Unendlichkeit (für den Anfang könntest du dir erst mal die Begriffe “abzählbar unendlich” und “überabzahlbar unendlich” anschauen).

    Ich hatte kürzlich gelesen, dass eine Hubblerate von 32 in Galaxienhaufen gefunden wurde. Wenn man da dann die gravitativen Gebundenheiten herausrechnet, was kommen für die Effekte der “dunklen Energie” dabei heraus?

    Sag mal, hatte ich dir nicht erst vor einigen Tage bei einem anderen Artikel erklärt, dass die Größe der Hubblerate nichts, aber auch gar nichts, mit der Dunklen Energie zu tun hat…?

  83. #84 Niels
    21. Juni 2011

    @Bjoern

    dass die Größe der Hubblerate nichts, aber auch gar nichts, mit der Dunklen Energie zu tun hat

    Wie meinen? Der Hubble-Parameter ist doch definiert als zeitliche Ableitung des Skalenfaktors geteilt durch den Skalenfaktor.
    Im Skalenfaktor ist doch zwangsläufig die Dunkle Energie drin, sogar als ganz wesentlicher Bestandteil.

  84. #85 Bjoern
    21. Juni 2011

    @Niels:

    Im Skalenfaktor ist doch zwangsläufig die Dunkle Energie drin, sogar als ganz wesentlicher Bestandteil.

    Meinst du, in der zeitlichen Entwicklung des Skalenfaktors, oder was?

    (für die zeitliche Entwicklung des Hubbleparameters ist die Dunkle Energie natürlich von Bedeutung – aber halt nicht für die derzeitige Größe des Hubbleparameters! Anders gesagt: die heutige Größe des Hubbleparameters und der heutige Wert der Dunklen Energiedichte sind zwei unabhängige Parameter des Modells)

  85. #86 Niels
    21. Juni 2011

    @Bjoern
    Nö, Dunkle Energiedichte und Hubble-Paramter sind keine unabhängigen Parameter. Auch nicht die heutigen Größen.
    Ganz kurz aus https://en.wikipedia.org/wiki/Lambda-CDM_model :

    “The ΛCDM model is based on six parameters: physical baryon density, physical dark matter density, dark energy density, scalar spectral index, curvature fluctuation amplitude and reionization optical depth. From these the other model values, including the Hubble constant and age of the universe, can be derived.”

  86. #87 Ralph Ulrich
    21. Juni 2011

    @Bjoen: “die Entstehung der Voids wird bereits durch die Dunkle Materie / Energie etc. erklärt. Warum ein neues Modell? Zweitens wiederhole ich meine Frage gestern: verstehe ich dich richtig? Du willst also eine Alternative zur Allgemeinen Relativitätstheorie entwickeln…?”
    Wo will ich die Entstehung der Voids erklären? Ich will die Raumstruktur dieser Gebiete erklären. Besser gesagt, ich will ganz allgemein im Sinne Einsteins erklären, was gekrümmter Raum ist, wie seine unterschiedliche Strukturen unterschiedliche Effekte bewirken.
    Physiker, die Gravitation und “dunkle Energie” als Kraft definieren, stellen sich doch gegen die Relativitätstheorie indem sie Gravitonen propagieren?

    Ich weiss nicht, wie ich mit Dir sprechen soll, ich habe kein Training im physikalischen Gespräch, da rutscht dann mal “Atom” durch, wenn ich natürlich AtomKERN meine. Ich komme mir vor wie ein Ausländer, der eine Story erzählen will, dem aber nicht zugehört wird, weil seine grammatikalischen Fehler stören.

    Bjoen: “Warum gibt es mehrere hunderte stabile Isotope”
    Soweit ich weiss, gibt es keine stabilen Isotope, sogar beim Wasserstoffkern spekuliert man, ob er nicht eine Halbwertzeit hat.

  87. #88 Alderamin
    21. Juni 2011

    @Ralph

    Wissenschaftler reagieren halt etwas allergisch, wenn jemand, der nich vom Fach ist, Theorien aufstellt. Ärzte mögen ja auch nicht besonders, wenn man mit einem Stapel von Wikipedia-Ausdrucken in ihre Praxis kommt. Fragen ist aber legitim.

    Ich will die Raumstruktur dieser Gebiete erklären. Besser gesagt, ich will ganz allgemein im Sinne Einsteins erklären, was gekrümmter Raum ist, wie seine unterschiedliche Strukturen unterschiedliche Effekte bewirken.

    Es gibt da ein schönes Gedankenexperiment. Dir dürfte bekannt sein, dass sich Objekte, die sich mit hoher Geschwindigkeit bewegen, nach der speziellen Relativitätstheorie in Bewegungsrichtung verkürzt erscheinen.

    Nach der allgemeinen Relativitätstheorie kann man nicht zwischen Schwerkraft und Beschleunigung unterscheiden. Jetzt denke Dir eine Zentrifuge, die sich sehr schnell dreht. Ein Objekt in der Zentrifuge erfährt eine Beschleunigung, weil jede Änderung der Bewegungsrichtung oder Geschwindigkeit eine Beschleunigung ist. Wenn die Beschleunigung aber nicht unterscheidbar ist von der Gravitation, müsste sie den Raum krümmen.

    Und das tut sie: denn durch die Rotation wird ein Objekt tangential gemäß der speziellen Relativitätstheorie verkürzt. Würde man also den Innenumfang der Zentrifuge messen, wenn sie sich sehr schnell dreht, dann würde ein kleinerer Wert als 2*Pi*Radius gemessen werden. Ein Kreis mit einem Radius von weniger als 2*Pi*r kann aber nur in einem gekrümmten Raum messbar sein (z.B. auf einer gebogenen Kugelfläche, da würde der Radius ja über der gekrümmten Oberfläche gemessen verlängert sein).

    So erklärt die Relativitätstheorie die Raumkrümmung. Die Vorhersage Einsteins war, dass auch die Masse den Raum krümmt, weil sie mit ihrer Gravitation eine Kraft erzeugt, die einer Beschleunigung entspricht. Und diese Krümmung lässt sich ja auch z.B. an der Lichtablenkung im Schwerefeld messen.

    Also: Der Raum ist flach, wo keine Masse ist, aber wo Masse ist, wird der Raum gekrümmt.

    Man kann an Strukturen in der kosmischen Hintergrundstrahlung ausmessen, dass das Weltall insgesamt flach ist, nur wo Massen sind, gibt es eine lokale Krümmung.

    Das Beispiel mit der Zentrifuge habe ich übrigens aus Brian Greenes Buch “Das elegante Universum”.

  88. #89 Bjoern
    21. Juni 2011

    @Niels:

    “The ΛCDM model is based on six parameters: physical baryon density, physical dark matter density, dark energy density, scalar spectral index, curvature fluctuation amplitude and reionization optical depth. From these the other model values, including the Hubble constant and age of the universe, can be derived.”

    O.k., das ist die Standard-Wahl für die Parameter. Genausogut könnte man aber auch andere Parameter als Grundgrößen wählen – und eine mögliche Wahl wäre H und Omega_Lambda (plus einige andere Größen) als unabhängige Parameter.

  89. #90 Alderamin
    21. Juni 2011

    Ein Kreis mit einem Radius von weniger als 2*Pi*r …

    Es musste hier natürlich Umfang heißen.

  90. #91 Bjoern
    21. Juni 2011

    @Ralph Ulrich:

    Ich will die Raumstruktur dieser Gebiete erklären. Besser gesagt, ich will ganz allgemein im Sinne Einsteins erklären, was gekrümmter Raum ist, wie seine unterschiedliche Strukturen unterschiedliche Effekte bewirken.

    Ich weiß nicht, was du mit “Raumstruktur” meinst, und ich sehe auch nicht, wie das, was du hier machst, “im Sinne Einsteins” ist…

    Physiker, die Gravitation und “dunkle Energie” als Kraft definieren, stellen sich doch gegen die Relativitätstheorie indem sie Gravitonen propagieren?

    Physiker definieren die dunkle Energie sicher nicht als Kraft, und die Gravitation auch nur dann, wenn sie klassisch rechnen (ohne allgemeine Relativitätstheorie). Und Gravitonen sind in keinster Weise widersprüchlich zur Relativitästheorie; Gravitonen sind einfach die Quanten, die zu Gravitationswellen (und ähnlichem) gehören, genauso wie Photonen die Quanten sind, die zu elektromagnetischen Wellen (und ähnlichem) gehören. Photonen sind doch auch kein Widerspruch zur klassischen Elektrodynamik – nur einer Erweiterung!

    Soweit ich weiss, gibt es keine stabilen Isotope, sogar beim Wasserstoffkern spekuliert man, ob er nicht eine Halbwertzeit hat.

    Der letzte Satzteil war richtig – das sind in der Tat Spekulationen! Wäre mir neu, dass irgend jemand bereits nachgewiesen hätte, dass es keine stabilen Isotope gibt…

  91. #92 Bjoern
    21. Juni 2011

    @Niels: Schau’ mal auf den Wiki-Artikel zu den Friedmann-Gleichungen:
    https://de.wikipedia.org/wiki/Friedmann-Gleichungen

    Kurz vor Schluss findet man die zeitliche Entwicklung des Skalenfaktors a(t). Und der hängt da halt nur von den Parametern Omega_Lambda und H_0 ab, die hier offensichtlich unabhängig voneinander sind.

  92. #93 Niels
    21. Juni 2011

    @Bjoern
    Die Formel ist mir bekannt.
    Schon witzig, dass der Sinus Hyperbolicus in guter Näherung die Expansion des Universums beschreibt, oder?

    Aber: Bloß weil in dieser Formel H_0 und Omega_lambda vorkommen, müssen die beiden Parameter doch nicht unabhängig voneinander sein. Die Materiedichte Omega_0 steht schließlich auch drin, obwohl einfach 1 – Omega_lambda = Omega_0 gilt, dieser Parameter also nicht unabhängig ist.
    H_0 könnte dort auch einfach aus Gründen der Einfachheit statt einem langen Term zur Berechnung von H_0 stehen.
    Schließlich stehen Dinge wie beispielsweise die Feinstrukturkonstante auch der Bequemlichkeit halber in vielen Formeln, obwohl sie sich herleiten lassen.

    Außerdem wird weiter oben im selben Artikel doch sogar Omega_lambda und Omega_0 mit Hilfe von H_0 definiert.
    https://upload.wikimedia.org/math/0/8/5/08593fbb7a73dfcfa927021eb3fecd70.png

    Das bedeutet für mich eigentlich schon, dass Omega_lambda von H_0 abhängig ist, oder?

    Genausogut könnte man aber auch andere Parameter als Grundgrößen wählen – und eine mögliche Wahl wäre H und Omega_Lambda (plus einige andere Größen) als unabhängige Parameter.

    Sicher, dass das geht?

  93. #94 Bjoern
    21. Juni 2011

    @Niels:

    Die Formel ist mir bekannt.
    Schon witzig, dass der Sinus Hyperbolicus in guter Näherung die Expansion des Universums beschreibt, oder?

    Hatten wir nicht vor ca. einem Jahr darüber schon mal geredet…? (vielleicht erinnere ich mich auch falsch, aber ich dachte, ich hätte das mit dem sinh dir damals sogar gesagt…)

    Aber: Bloß weil in dieser Formel H_0 und Omega_lambda vorkommen, müssen die beiden Parameter doch nicht unabhängig voneinander sein. Die Materiedichte Omega_0 steht schließlich auch drin, obwohl einfach 1 – Omega_lambda = Omega_0 gilt, dieser Parameter also nicht unabhängig ist.

    Omega_0 ist nur dann ein abhängiger Parameter, wenn man ein geschlossenes Universum voraus setzt – sondern ist der Parameter auch unabhängig, würde ich sagen.

    H_0 könnte dort auch einfach aus Gründen der Einfachheit statt einem langen Term zur Berechnung von H_0 stehen.

    Selbst wenn das so wäre – Fakt ist doch, dass laut dieser Formel die zeitliche Entwicklung des Skalenfaktors eindeutig festgelegt ist, wenn man H_0 und Omega_Lambda kennt. (und selbst wenn man H_0 berechnen kann, dann sicher nicht nur aus Kenntnis von Omega_Lambda, sondern man bräuchte wieder mindestens eine andere zusätzliche Größe – warum also nicht gleich H_0 als unabhängige Größe nehmen?)

    Schließlich stehen Dinge wie beispielsweise die Feinstrukturkonstante auch der Bequemlichkeit halber in vielen Formeln, obwohl sie sich herleiten lassen.

    Auch da könnte man sich streiten – was ist der fundamentale unabhängige Parameter? Die Feinstrukturkonstante – oder die Elektronenladung? (plus hquer und c) Ich als Teilchenphysiker würde die Feinstrukturkonstante als fundamentaler betrachten…

    Außerdem wird weiter oben im selben Artikel doch sogar Omega_lambda und Omega_0 mit Hilfe von H_0 definiert.

    Wieder: ob wir jetzt rho_0 und Lambda oder Omega_0 und Omega_Lambda als fundamentale Parameter nehmen, ist doch letztlich wurscht!

    Das bedeutet für mich eigentlich schon, dass Omega_lambda von H_0 abhängig ist, oder?

    Ob Größe A von Größe B abhängt, kann man IMO schlecht allgemein sagen – man muss schon dazu sagen, welche Größen C, D, E, … dabei konstant gehalten werden… (für konstantes Lambda (und ich meine hier jetzt nicht “zeitlich konstant”, sondern “im Vergleich verschiedener Modelle konstant”) hängt Omega_lambda z. B. natürlich von H_0 ab)

  94. #95 Niels
    21. Juni 2011

    @Bjoern
    Hatten wir nicht vor ca. einem Jahr darüber schon mal geredet…?
    Stimmt, du hast mir das mit dem sinh erzählt und mich dazu angeregt, das alles mal ordentlich in Mathematica zu berechnen, einschließlich Ereignishorizont, Hubble-Radius, Partikelhorizont usw.
    https://www.scienceblogs.de/astrodicticum-simplex/2010/10/klingeltone-aus-dem-schwarzen-loch.php
    Sorry, wusste nicht mehr, das du das warst. Nochmal ein Danke schön nachträglich.

    Omega_0 ist nur dann ein abhängiger Parameter, wenn man ein geschlossenes Universum voraus setzt – sondern ist der Parameter auch unabhängig, würde ich sagen.
    Meinst du wirklich geschlossen?
    1 – Omega_lambda = Omega_0 gilt doch nur bei Krümmung Null, also flach und damit offen.
    Die Formel mit dem sinh gilt doch nach Herleitung auch nur bei Krümmung Null. Sonst fällt der 1/(a^2) – Beitrag nicht weg.

    Über die Sache mit den Parametern muss ich nochmal nachdenken. Wahrscheinlich hast du aber recht.

  95. #96 Ralph Ulrich
    21. Juni 2011

    @Alderamin, schönes Beispiel die Zentrifuge. Hatte ich vor Jahren bei Brian Greene wohl nicht behalten, weil soviel neu in dem Buch für mich war. Oder ich habe unbewußt selectiv gelesen. Jedenfalls widerspricht es meinem Bild (Theorie will lieber nicht sagen, eher Idee) von “gekrümmter Raumstruktur”.

    Ich lese Physiker Texte wohl auch deswegen selektiv, weil ich nicht alles verstehen kann. So auch mal über kurz ausgerollte (nicht aufgerollt) zusätzliche Dimensionen bei:
    https://de.wikipedia.org/wiki/Lisa_Randall
    Was mir “selektiv” entgegenkam. Denn seit ich (schon) über Stringtheorien mit 11 oder mehr Dimensionen hörte, mache ich mir Gedanken über die “Natur” von Dimensionen. Und eigentlich fing es mit einer kurzen Nachricht so ungefähr 1994 an, in der es hieß, dass jemand ausgerechnet hat, das die Zentrifugalkraft sich hinter dem Ereignishorizonts eines scharzen Lochs umkehrt. Was ist daraus eigentlich geworden: Wenn etwas hinter dem Ereignishorizont beschleuningt wird, dann zieht es nicht nach außen sonder verstärkt durch dies Beschleunigung in Richtung Zentrum des scharzen Lochs.
    Ist das weiter bestätigt worden?

    Jedenfalls habe ich mir dann Universen in scharzen Löchern gedacht, wo also der Raum im schwarzen Loch sich erweitert. Ohne von Multiversen Theorien gelesen zu haben.

    Dann kam fünf Jahre später mir ein Text über Stringtheorie vor Augen. Da ich schon Training im Denken von versteckten Dimensionen hatte, dachte ich mir einen Sinn von Dimensionen aus:

    Nachbarschaften (Mathematiker würden wohl von Freiheitsgraden sprechen):
    Anzahl_der_Nachbarschaften_mit_sich_selbst=3**Dimensionen

    Ich habe diese Formel einfach von den Schritten Punktuniversum zu Gerade zu Flächenuniversum abgeleitet, um eine Rechenformel zu haben mir das Unvorstellbare darzustellen, was ein Universum mit mehr als zehn Dimensionen bedeutet. Das “aufgerollte” Dimensionen der Stringtheorie keine Bedeutung in ihren Auswirkungen hätten, wurde mir erst durch Lisa Randall gesagt.

    Jedenfalls denke ich seit dem, warum setzt sich kein geometrisch begabter Physiker daran eine geometrisch begründete Theorie der Schwerkraft zu entwickeln?

    Natürlich würden 1 Milliarde zu berechnender Strings in zehn oder mehr Dimensionen innerhalb eines Quarks auch heute noch die Rechenkapazitäten von Supercomputern sprengen …

  96. #97 Ralph Ulrich
    21. Juni 2011

    Materie ist in meiner Idee der Realität ohne bleibende Identität: ruhende Raumstrings werden durch Schwingungsansteckung benachbarter Materiestrings zu eben dieser Materie. Bewegung eines Objekts wäre dann einfach eine Raumschwingung, die weiter getragen wird. Gravitation entsteht dadurch, dass in Richtung schwerer Objekte mehr Raumstrings zur Verfügung stehen, weil Masse zusätzliche Dimensionen der aufgerollten Dimensionen der Raumstrings entrollt. Diese brauche Nachbarn, weswegen in Richtung Masse mehr Raum zur Verfügung steht. Gravitation ist in ihrer Wirkung dadurch nur ein Gleiten von vorbeifliegender Masse in Richtung des statistischen Durchnitts des zur Verfügung stehenden Raumes.
    … sicher reichlich kurz und unbeholfen formuliert, aber für mich im Kopf so anschaulich, dass ich auch durch ein Zentrifugengedankenexperiment nicht so einfach davon loskomme. Ich weiss nämlich seit Kuhn und Feyerabend, dass andere Paradigmen tiefgreifende Umstellungen der Beobachtungsbegriffe brauchen (Giordano Bruno hatte wissenschaftlich nicht Recht in seiner Zeit!).

  97. #98 Florian Freistetter
    22. Juni 2011

    @Ralph Ulrich: “Materie ist in meiner Idee der Realität ohne bleibende Identität: ruhende Raumstrings werden durch Schwingungsansteckung benachbarter Materiestrings zu eben dieser Materie”

    Bitte was??

  98. #99 Bjoern
    22. Juni 2011

    @Niels:

    Meinst du wirklich geschlossen?

    Ups, ‘tschuldigung, meinte natürlich flach.

  99. #100 Bjoern
    22. Juni 2011

    @Ralph Ulrich:

    Ich lese Physiker Texte wohl auch deswegen selektiv, weil ich nicht alles verstehen kann. So auch mal über kurz ausgerollte (nicht aufgerollt) zusätzliche Dimensionen bei:
    https://de.wikipedia.org/wiki/Lisa_Randall

    Ah ja – einiges, was da steht, klingt ja ganz ähnlich wie bei dir… (im Wiki-Artikel ergibt’s aber teilweise auch nicht viel Sinn; ich habe das Gefühl, den Teil hat jemand geschrieben, der Randalls Ideen teilweise selbst nicht so ganz kapiert hat…)

    Nachbarschaften (Mathematiker würden wohl von Freiheitsgraden sprechen):
    Anzahl_der_Nachbarschaften_mit_sich_selbst=3**Dimensionen

    Den Sinn dieser Formel verstehe ich weder mit “Nachbarschaften” noch mit “Freiheitsgraden”. (kurze Frage: ** steht bei dir für potenzieren, oder? wie in einigen Programmiersprachen)

    Ich habe diese Formel einfach von den Schritten Punktuniversum zu Gerade zu Flächenuniversum abgeleitet,…

    Könntest du diese Fälle vielleicht kurz mal demonstrieren? Hoffentlich wird dann die Formel klar…

    Jedenfalls denke ich seit dem, warum setzt sich kein geometrisch begabter Physiker daran eine geometrisch begründete Theorie der Schwerkraft zu entwickeln?

    ????? Schon mal was von Einstein und der Allgemeinen Relativitätstheorie gehört ?????

    Natürlich würden 1 Milliarde zu berechnender Strings in zehn oder mehr Dimensionen innerhalb eines Quarks auch heute noch die Rechenkapazitäten von Supercomputern sprengen …

    Nach der derzeitigen Stringtheorie besteht ein Quark doch aus genau einem String, nicht aus 1 Milliarde…?

    Materie ist in meiner Idee der Realität ohne bleibende Identität: ruhende Raumstrings werden durch Schwingungsansteckung benachbarter Materiestrings zu eben dieser Materie.

    “Schwingungsansteckung”? Was ist das, und wie funktioniert das?

    Bewegung eines Objekts wäre dann einfach eine Raumschwingung, die weiter getragen wird.

    Ich nehme an, damit meinst du eine Welle? (dir ist schon klar, dass eine Welle nichts anderes als eine sich im Raum ausbreitende Schwingung ist – oder?)

    Gravitation entsteht dadurch, dass in Richtung schwerer Objekte mehr Raumstrings zur Verfügung stehen, weil Masse zusätzliche Dimensionen der aufgerollten Dimensionen der Raumstrings entrollt.

    (1) Wie und warum sollte sie das tun? (2) Wie folgt daraus, dass mehr “Raumstrings zur Verfügung stehen”?

    Diese brauche Nachbarn, weswegen in Richtung Masse mehr Raum zur Verfügung steht.

    Und warum sieht man diesen “mehr Raum” nicht bzw. kann diesen nicht direkt nachmessen?

  100. #101 Ralph Ulrich
    22. Juni 2011

    ‘**’ steht natürlich für das Potenzieren. Das Herleiten meiner Formel war ganz einfach: 3**0=1 im Punktuniversum kann ich nur auf meinem Fleck bleiben. Auf einer Linie 3**1=3 kann ich noch zusätzlich vor und zurück. Auf einer Fläche noch zusätzlich seitwärts uns schräg. Das gibt einen guten Eindruck für ein zehndimensionales Universum: 59049 Bewegungsmöglichkeiten. Ich glaube es gibt auch Stringtheorien mit Dimensionen:
    3**27=7.625.597.484.987
    7 Billarden Bewegungsmöglichkeiten hätten wir in einem Universum, auch wenn alle Dimensionen nur kurz wären (bei Lisa Randall ist eine zusätzliche Dimension ausgerollt). Diese Formel ist natürlich nur gut um sich die potentiellen Möglichkeiten von Dimensionen vor Augen zu führen, denn die Formel geht von einer ganz primitiven quaderförmigen, wabenförmigen Struktur aus.

    Ein String, der ruhenden Raum darstellt, müsste eine Planckgröße haben. Deswegen kam ich darauf, dass Quarks aus Milliarden von Strings bestehen müssten. Da der Raumstring ruhend ist, hat er nicht die Kraft zusätzliche Dimensionen auszurollen. Anders ist es, wenn ein Materieteilchen vorbeikommt. Da es “in Wirklichkeit” keinerlei Bewegung von Materie gibt, übertragen die Materie-Energie-geladenen Raumteilchen ihre Energie auf den nächsten Raumstring in der Bewegung!

    Die Materie-Energie ist einfach eine Schwingung in zusätzlichen Dimensionen. Welche Dimensionen dabei ausgerollt werden, bestimmt die Qualitäten der Materie: Spin, Ladung, “Farbe” der Quarks etc. Die Schwere wird durch die Anzahl der ausgerollten Dimensionen bestimmt.

    Die Trägheit der Masse bei Beschleunigung ist der Aufwand an Kraft und Energie die Schwingungen in den Dimensionen zu “synchronisieren”, das heisst in eine gemeinsame Richtung zu bringen, damit Bewegung entsteht.

    Die zusätzlichen Andockmöglichkeiten (siehe meine Nachbarschaftsformel) von Materie bewirken bei “ruhenden” 3dim Raumstrings aber, dass sie quasi “virtuell” kurzzeitige Ausrollungen der entsprechenden Dimensionen produzieren können, und sich also zusätzlich andocken an Materie, weswegen also in der Nähe von schweren Objekten “dichterer”, gekrümmter Raum entsteht. Gravitationswirkung ist dann nur eine in die statistische Mitte zielende Bewegungsschwingungs …

    … aber das Zentrifugengedankenexperiment bringt mich bei meinen Ideen etwas aus dem Konzept …

  101. #102 Bjoern
    22. Juni 2011

    @Ralph Ulrich:

    Auf einer Linie 3**1=3 kann ich noch zusätzlich vor und zurück.

    Da werden also nicht nur die Nachbarn gezählt, sondern auch der Punkt selbst?

    Auf einer Fläche noch zusätzlich seitwärts uns schräg.

    Du setzt also ein quadratisches Gitter voraus, bzw. in 3D dann ein Würfel-Gitter? (ah ja, sehe gerade, das sagst du weiter unten selbst)

    Außerdem sind (in einem quadratischen Gitter) diese “Bewegungsmöglichkeiten” nicht unabhängig voneinander – eine Bewegung nach z. B. rechts oben ist zusammengesetzt aus einer Bewegung nach rechts und einer nach oben. Warum sollte man die also extra zählen?

    Ich glaube es gibt auch Stringtheorien mit Dimensionen:
    3**27=7.625.597.484.987

    Vor “Dimensionen” fehlt hier wohl die Zahl 27? Wäre mir übrigens neu, dass es solche Stringtheorien gibt. Soweit ich weiss, gibt es eine mit 26 Dimensionen; aus irgendwelchen komplizierteren Gründen, an die ich mich auch nicht genau erinnere (hat, glaube ich, was mit Supersymmetrie zu tun) funktioniert die aber nicht, sondern man braucht die übliche mit 10 Dimensionen.

    7 Billarden Bewegungsmöglichkeiten hätten wir in einem Universum, auch wenn alle Dimensionen nur kurz wären (bei Lisa Randall ist eine zusätzliche Dimension ausgerollt).

    Und bei Lisa Randall (und meines Wissens auch in der üblichen Stringtheorie) steckt natürlich explizit mit drin, dass normale Teilchen an unseren 3D-Raum gebunden sind und sich in den anderen Richtungen gar nicht bewegen können.

    Ein String, der ruhenden Raum darstellt, …

    Ruhend in Bezug auf was?

    …müsste eine Planckgröße haben.

    Warum?

    Da der Raumstring ruhend ist, hat er nicht die Kraft zusätzliche Dimensionen auszurollen.

    ???

    Da es “in Wirklichkeit” keinerlei Bewegung von Materie gibt, übertragen die Materie-Energie-geladenen Raumteilchen ihre Energie auf den nächsten Raumstring in der Bewegung!

    Wie funktioniert diese Übertragung? Eine Übertragung von Energie setzt eine Kraft voraus. Üben die “Raumstrings” also Kräfte aufeinander aus?

    Die Materie-Energie ist einfach eine Schwingung in zusätzlichen Dimensionen. Welche Dimensionen dabei ausgerollt werden, bestimmt die Qualitäten der Materie: Spin, Ladung, “Farbe” der Quarks etc. Die Schwere wird durch die Anzahl der ausgerollten Dimensionen bestimmt.

    Na, das nenne ich mal eine wilde, durch nichts belegte, Spekulation… Warum nochmal sollte sich irgend jemand die Mühe machen, das irgendwie mathematisch nachzuvollziehen…?

    Die Trägheit der Masse bei Beschleunigung ist der Aufwand an Kraft und Energie die Schwingungen in den Dimensionen zu “synchronisieren”, das heisst in eine gemeinsame Richtung zu bringen, damit Bewegung entsteht.

    Warum ist für diese “Synchronisation” Energie nötig? Du scheinst voraus zu setzen, dass die “Raumstrings” ihrerseits wieder eine Trägheit haben – wenn sie diese nicht hätten, wäre nämlich logischerweise keinerlei Energie nötig, um sie in Bewegung zu setzen…

    …weswegen also in der Nähe von schweren Objekten “dichterer”, gekrümmter Raum entsteht.

    Du hast immer noch nicht erklärt, was “dichter Raum” überhaupt heißen soll… (ich habe so die vage Ahnung, dass du damit meinst, “mehr Raum in ein und demselben Volumen”; dass das ein Widerspruch in sich wäre, macht ja nichts…)

  102. #103 Ralph Ulrich
    22. Juni 2011

    @Bjoern,

    die Einschränkung der Bewegung auf hoch/runter ist zu Roboterhaft. Wenn die Qualität der Ästhetik einer Formel etwas über den Realitätsgehalt sagen kann, dann gibt es auch schräge Schritte zum Nachbarpunkt.

    Ich wusste, dass du sagen wirst: “sehe gerade, das sagst du weiter unten selbst”. So kann man also Dir Wind aus den Segeln nehmen mit kleinen Einschränkungen, mit juristischen Klauseln der beschränkten Haftung. Ah! Aber eigentlich glaube ich an die volle Wirksamkeit der Formel in Punkto Freiheitsgrade der Bewegung! Und dabei bitte noch im Kopf behalten, dass diese Nachbarschaften der zusätzlichen Dimensionen Abkürzungen der Wege in drei Dimensionen sind (Das würdest auch Du verstehen, wenn Du Dich auf das Beispiel des geknickten Blatt Papiers einlassen könntest): Die Nachbarn sind in unserer normalen 3dim Welt unter Umständen sehr weit entfernt!

    Lisa Randall hat mit ihrer Behauptung, dass es eine zusätzliche ausgerollte, wenn auch kurze Dimension gibt, mich nur “selektiv” bestätigt in meinen Ideen. Sonst habe ich damit nichts am Hut. By the way: Mir ginge die physikalische Kompetenz ab bei ihr auch nur “abzuschreiben”.

    Raumstrings/Raumquanten müssten Planckgröße habe, sonst hätten wir keine kontinuierlichen Messergebnisse in den kleinen Raumeinheiten.

    Bjoern : “wilde, durch nichts belegte, Spekulation”
    Ja, nur Ideen, und mein Wundern, dass es in dieser Richtung in der Physik bisher anscheinend nichts an Forschung gibt. Aber wie hier schon gesagt wurde, es gibt wohl tausende solcher Ideengebäude …

    Bjoern: “Du scheinst voraus zu setzen, dass die “Raumstrings” ihrerseits wieder eine Trägheit haben”
    Die Raumstrings sind Materiestrings, wenn sie stärker und auch in zusätzlichen Dimensionen schwingen, die “aufgerollten” Dimensionen der Stringtheorie ein wenig ausrollen. Jede Schwingung ist Träge, wenn es darum geht ihre Richtung ändern zu wollen.

    Ich glaube auch, dass die Schwingungen der Strings in Materie den Zeitablauf bestimmen, und mit dieser Zeit auch ihre Zerfallswahrscheinlichkeit, siehe oben. Bei Bewegung im Raum wird durch die Notwendigkeit der Übertragung der Materieschwingung auf nachbarliche Raumstrings der Materie ein wenig von der Zeit der inner-Materieschwingung genommen. Ich glaube so könnte man Zeitdilatationen in Richtung Lichtgeschwindigkeit erklären. Auch die Idee, dass an großen Massen mehr Raumstrings pro Volumen existieren, harmoniert mit der Zeitänderung an großen Massen in meinem Modell: inner-Materie Schwingungen werden sozusagen abgelenkt und aufgebraucht durch zusätzliche, angrenzende Raumstrings.

    Dichteren Raum kann man sich nur so vorstellen: Eine gleichmäßige Wabenstruktur, die im dichteren Fall an großen Massen durch zusätzliche hineingequetschte Waben gekennzeichnet ist: non-fitting war mein Ausdruck dafür.

  103. #104 Ralph Ulrich
    22. Juni 2011

    Es ist von meiner Idee her gar nicht festgelegt, ob alle Strings im Volumen eines Quarks sich durch Materieschwingung auszeichnen. Vielleicht gibt es auch viele ruhenden Raumstrings innerhalb von Baryonen.

  104. #105 Bjoern
    22. Juni 2011

    @Ralph Ulrich:

    die Einschränkung der Bewegung auf hoch/runter ist zu Roboterhaft.

    Äh, nein, das ist eine simple mathematische Betrachtung (nennt sich “lineare Unabhängigkeit”). Du wolltest doch eine mathematische Behandlung deiner Ideen – und wenn’s jemand macht, dann ist dir nicht recht, was die Mathematik dazu sagt…?

    Wenn die Qualität der Ästhetik einer Formel etwas über den Realitätsgehalt sagen kann, dann gibt es auch schräge Schritte zum Nachbarpunkt.

    ‘tschuldigung, aber “Qualität” und “Ästhetik” sind keine Kriterien für Realitätsgehalt. (kann höchstens vielleicht dann als Entscheidungshilfe benutzt werden, wenn man mehrere Theorien hat, die absolut gleichwertige Vorhersagen liefern, was hier eindeutig nicht der Fall ist)

    Und dabei bitte noch im Kopf behalten, dass diese Nachbarschaften der zusätzlichen Dimensionen Abkürzungen der Wege in drei Dimensionen sind (Das würdest auch Du verstehen, wenn Du Dich auf das Beispiel des geknickten Blatt Papiers einlassen könntest): Die Nachbarn sind in unserer normalen 3dim Welt unter Umständen sehr weit entfernt!

    Wenn man die Analogie mit dem geknickten Blatt Papier benutzt, dann gibt es manche Nachbarn, die in unserer normalen Welt sehr weit entfernt sind – für die weitaus überwiegende Anzahl an Nachbarn gilt das nicht!

    Die Raumstrings sind Materiestrings, wenn sie stärker und auch in zusätzlichen Dimensionen schwingen, die “aufgerollten” Dimensionen der Stringtheorie ein wenig ausrollen. Jede Schwingung ist Träge, wenn es darum geht ihre Richtung ändern zu wollen.

    Jede (mechanische) Schwingung ist deshalb träge, weil die Teilchen, die da schwingen, von sich aus eine Massenträgheit haben. Letztlich versuchst du hier also Massenträgheit durch Massenträgheit zu erklären…

    Ich glaube auch, dass die Schwingungen der Strings in Materie den Zeitablauf bestimmen, und mit dieser Zeit auch ihre Zerfallswahrscheinlichkeit, siehe oben. Bei Bewegung im Raum wird durch die Notwendigkeit der Übertragung der Materieschwingung auf nachbarliche Raumstrings der Materie ein wenig von der Zeit der inner-Materieschwingung genommen. Ich glaube so könnte man Zeitdilatationen in Richtung Lichtgeschwindigkeit erklären.

    ‘tschuldigung, aber das ergibt mal wieder von vorne bis hinten keinen Sinn. Fangen wir mal ganz langsam an: eine Schwingung ist eine Bewegung, das in der Zeit stattfindet – ohne Zeit kann man keine Schwingung haben. Wie können dann die Schwingungen ihrerseits den Zeitablauf bestimme?!? Da beisst sich doch die Katze in den Schwanz!

    Auch die Idee, dass an großen Massen mehr Raumstrings pro Volumen existieren,…

    Das ergibt keinen Sinn. “Raumstrings” setzen laut dir ja den Raum zusammen. Wenn man mehr Raumstrings hat, dann ist also mehr Raum da, also ist automatisch das Volumen größer (zur Erinnerung: “Volumen” ist dasselbe wie “Rauminhalt”, sprich: ein Mass dafür, wie viel Raum da ist!) Die Anzahl der Raumstrings pro Volumen muss also defintionsgemäß immer exakt gleich groß sein!

    Du bist nicht konsequent: wenn der Raum aus Raumstrings besteht, dann heißt “mehr Raumstrings” automatisch, dass mehr Raum da ist – und was soll denn “mehr Raum” anderes heißen als “mehr Volumen”?!? Du dagegen betrachtest jedes Mal, wenn du von “dichterem” bzw. “dünneren” Raum redest, deine “Raumstrings” nicht wirklich als etwas, aus denen der Raum besteht – sondern als etwas, das im Raum existiert und diesen ausfüllt!

    Dichteren Raum kann man sich nur so vorstellen: Eine gleichmäßige Wabenstruktur, die im dichteren Fall an großen Massen durch zusätzliche hineingequetschte Waben gekennzeichnet ist:…

    Damit man Waben “hineinquetschen” kann, müssten diese kleiner sein als die anderen. Kleiner als die Plancklänge geht aber nicht… (wenn du die Raumquantisierung ernst nimmst – wenn du sie dagegen nicht ernst nimmst, dann ergibt dein ganzes Konzept von Nachbarn etc. sowieso keinen Sinn)

  105. #106 Bjoern
    22. Juni 2011

    @Ralph Ulrich: Ach, und noch etwas zu deinen “Bewegungsmöglichkeiten”: man ausrechnen, dass pro Bewegungsmöglichkeit (pro “Freiheitsgrad”) ein Teilchen bei konstanter Temperatur T jeweils die mittlere Energie 0,5 * k * T hat (k ist die Boltzmann-Konstante). Und man kann diese Energie solcher Teilchen auch nachmessen; Ergebnis: die mittlere Energie pro Teilchen ist 1,5 * k * T. Offensichtliche Folgerung: jedes Teilchen hat 3 Freiheitsgrade, also 3 Bewegunsmöglichkeiten (rauf-runter, links-rechts, vor-zurück); schräge Bewegungen tragen da nichts bei…

  106. #107 Bjoern
    22. Juni 2011

    @Ralph Ulrich: Und noch etwas zum Nachbarn-Konzept. Dass die “schrägen” Nachbarn einen größeren Abstand zum Punkt haben als die “direkten” Nachbarn, stört dich nicht? Ist d die Anzahl der Dimensionen und a der Abstand zu einem direkten Nachbarn, dann ist der Abstand zu einem “schrägen Nachbarn” (ein quadratisches Gitter vorausgesetzt) gleich Wurzel(d) * a. Schon für d > 4 ergibt das Werte größer als 2a, also sogar größer als der Abstand zum übernächsten (!) Nachbarn in “gerader” Linie…

  107. #108 Ralph Ulrich
    22. Juni 2011

    @Bjoern, Wollte nur kurz sagen, dass diese:
    “Eine Rose ist eine Rose”
    Definitionsprobleme meiner Ideen die großen Knackpunkte sind. Vielleicht endet jede ins tiefste gehende Theorie zwangsläufig an so einem Endpunkt?
    Darüber muss ich noch nachdenken, bin gerade auf Arbeit.

  108. #109 Ralph Ulrich
    23. Juni 2011

    @Bjoern: Man kann wirklich per Temperatur ausmessen, wie sich Quanten bewegen? Danach haben Dimensionen also diese Auswirkung auf die Freiheitsgrade: F=1+2*Dim ? Das wäre um einige Größenordnungen weniger als ich dachte.

  109. #110 Bjoern
    23. Juni 2011

    @Ralph Ulrich:

    Man kann wirklich per Temperatur ausmessen, wie sich Quanten bewegen?

    Ich habe von “Teilchen” geredet (genauer: von den Atomen einatomiger Gase wie z. B. Helium), nicht von “Quanten”. Und man kann “per Temperatur ausmessen, wie sie sich bewegen”, sondern man kann ihre Energie in Abhängigkeit von der Temperatur messen und daraus schließen, wie viele Bewegungsmöglichkeiten (Freiheitsgrade) sie haben.

    Danach haben Dimensionen also diese Auswirkung auf die Freiheitsgrade: F=1+2*Dim ?

    Ich weiß nicht, wo diese Formel schon wieder herkommt. Korrekt wäre schlicht und einfach (für einatomige Gase): Anzahl der Freiheitsgrade = Anzahl der (räumlichen) Dimensionen.

  110. #111 Ralph Ulrich
    24. Juni 2011

    Mein Formel ist die gleiche, nur dass ich mit plus Eins die Freiheit auf dem Eigenpunkt zu bleiben mitzähle, und die Nachbarschaften mal zwei für jede Seite, denn es wäre ja nun doof mit 0,5 zählen zu müssen, falls die Stringtheoretiker auf den Trichter kämen, dass eine Dimension nur zu einer Seite ausgerollt wäre.

    Ganz allgemein muss es nicht:
    1. so sein, dass Quanten, oder gar sub-Strings von Quanten, sich so verhalten, wie ganze Atome. Also vielleicht doch auch schräge wandern können, denn es
    2. muss nicht so sein, dass ein aus Raumstrings aufgebauter Raum eine Wabenstruktur von Quadern hat. Es gibt noch Oktoeder (oder hießen diese Teile Dodekaeder?), die sich wabenförmig ohne Lücken in einen Raum einpassen können. Dann gibt es auch Schrägen, die nicht dreiecksmäßig hypothenusig länger sind!

    Also man kann nur sagen, ein zehndimensionaler Raum (vollständig ausgerollter Dimensionen) hält mindestens 21 Freiheiten die Position zu ändern vor. Vielleicht aber auch 50 tausend.

  111. #112 Ralph Ulrich
    24. Juni 2011

    Nachtrag: Die Eins in meiner Formel ist wichtig für das Punkuniversum. Ohne die Eins wäre jede Existenz im nächsten Moment verschwunden. Es könnte keinen Urknall geben.

  112. #113 rolak
    24. Juni 2011

    Hey, das klingt interessant: Wo muß ich abbiegen, um zum Punkuniversum zu kommen? Hinter dem kleineren Mond von Bogosity7 links?
    =>Freiheitsgrad, Ralph Ulrich. Es gibt übrigens Eigenwerte und -vektoren, jedoch keine -punkte.

  113. #114 Bjoern
    24. Juni 2011

    @Ralph Ulrich:

    Die Eins in meiner Formel ist wichtig für das Punkuniversum. Ohne die Eins wäre jede Existenz im nächsten Moment verschwunden.

    Äh, nein. Zur Erinnerung: die Freiheitsgrade zählen die Bewegungsmöglichkeiten. Und dass es in einem Universum, dass nur aus einem Punkt besteht, exakt null Möglichkeiten gibt, sich irgendwo hinzubewegen, ist ja wohl irgendwie logisch…

    …und die Nachbarschaften mal zwei für jede Seite, denn es wäre ja nun doof mit 0,5 zählen zu müssen, falls die Stringtheoretiker auf den Trichter kämen, dass eine Dimension nur zu einer Seite ausgerollt wäre.

    Wenn das so wäre, dann hätte man nur am Rand jeweils nur einen “Nachbarn”, mittendrin gäbe es immer noch zwei Nachbarn. Also wäre der Faktor 0,5 da auch nicht nötig.

    Ganz allgemein muss es nicht:
    1. so sein, dass Quanten, oder gar sub-Strings von Quanten, sich so verhalten, wie ganze Atome.

    (1) Erst mal verhalten sich Atome unter den meisten Bedingungen sowieso schon selbst als Quanten, also ist das ein seltsamer Vergleich. (2) Schon mal was von “special pleading” gehört?

    Also man kann nur sagen, ein zehndimensionaler Raum (vollständig ausgerollter Dimensionen) hält mindestens 21 Freiheiten die Position zu ändern vor.

    Nein, 10. Dass du keine Mathematik (und Physik) magst und kannst, ändert nichts an den Tatsachen.

  114. #115 Ralph Ulrich
    24. Juni 2011

    @Bjoern, mathematisch hast Du jetzt die Eins in ihrer physikalischen Auswirkung beim Urknall nicht verstanden:
    Mögliche Position im nächsten Moment (ohne eins) = Null Dimensionen * 2.
    Nichts wäre mehr vorhanden ohne die eins in der Formel.

    Ich will mir mit meiner Formel einen Eindruck verschaffen, wieviele Nachbarn angedockt sein könnten an zusätzlichen Dimensionen, deswegen ist mir diese Verdoppelung wichtig: Mindestens mal zwei! Wenn die Wabenstruktur anders wäre als nur mit Quadern, käme man doch auf höhere Größenordnungen (3**Dimensionen).

  115. #116 Ralph Ulrich
    24. Juni 2011

    Nochmal zur gemessenen Freiheit der Bewegung von Teilchen: Wenn bestimmte Eigenschaften von Materie, wie Spin, “Farbe” oder Ladung durch Schwingungen der Strings in bestimmten Raumdimensionen gekennzeichnet sind, kann eine Möglichkeit der Bewegung durch eben diese Eigenschaften der Schwingung eingegrenzt sein, was ein “quer” verbietet. Das muss also noch nichts aussagen über die Möglichkeiten des Raumes.

  116. #117 Bjoern
    24. Juni 2011

    @Ralph Ulrich:

    mathematisch hast Du jetzt die Eins in ihrer physikalischen Auswirkung beim Urknall nicht verstanden:

    Ein Universum, das punktförmig ist und immer sein wird, dehnt sich eh nicht aus – also ergibt es da auch keinen Sinn, von einem Urknall zu reden.

    Ich will mir mit meiner Formel einen Eindruck verschaffen, wieviele Nachbarn angedockt sein könnten an zusätzlichen Dimensionen,

    Wenn du mit deiner Formel die Anzahl der Nachbarn zählen willst, dann ergibt es keinen Sinn, den Punkt selbst mitzuzählen. Wenn du die Anzahl der Freiheitsgrade zählen willst – die ist, wie gesagt, einfach gleich der Anzahl der (räumlichen) Dimensionen.

    Und außerdem habe ich dir im Kommentar vorgestern, 13:50 Uhr, noch einen weiteren Grund genannt, warum es wenig Sinn ergibt, die “schrägen” Nachbarn mitzuzählen – auf den bist du bisher überhaupt nicht eingegangen.

    Wenn die Wabenstruktur anders wäre als nur mit Quadern, käme man doch auf höhere Größenordnungen (3**Dimensionen).

    Ja, wenn… Gibt es irgend einen Grund, das zu vermuten?

    …kann eine Möglichkeit der Bewegung durch eben diese Eigenschaften der Schwingung eingegrenzt sein,…

    Ich wüsste nicht, wie.

  117. #118 Ralph Ulrich
    24. Juni 2011

    Bjoern: “Ja, wenn… Gibt es irgend einen Grund, das zu vermuten?”
    Was wird den aus einer Menge von Tischtennisbällen, die in eine Abfallpresse geraten: perfekte Quader, oder doch eher etwas Kugel ähnlicheres?

    Bei der Einschränkung der Bewegungsfreiheit habe ich jetzt nur das Bild im Kopf eines kreiselnden Sateliten, der aus eben dem Grund der Bewegungsverhinderung kreiseln soll. Ich weiss jetzt aber nicht, ob man kreiseln mit schwingen physikalisch vergleichen kann. Aber wer weiss, vielleicht sind ja Bewegungen, die wir in unserer dreidimensionalen Welt als Schwingungen sehen, in Wirklichkeit Kreisbewegungen durch zusätzliche Dimensionen hindurch.

  118. #119 stefan
    24. Juni 2011

    Ich glaube die dunkle Materie gibt es nicht, denn man geht davon aus das ein
    Schwarzes Loch einfach so stark ist das es Licht anzieht aber es ist ca. 6 mal
    so Stark (dunkler) als angenommen.
    Und das hält die Galaxie zusammen, denke Ich.
    Ausserdem ist ein Schwarzes Loch so eine art grosser Recycler des Universums
    und die Verbindung in ein anderes Universum, in dem der Kreislauf der Energie
    fortgesetzt wird.

    Jedes grössere Universum Hat ein Schwarzes Loch, weil die dichte irgendwann so
    gross wird das kein Licht mehr nach aussen dringt.
    Mfg Steff

  119. #120 Florian Freistetter
    24. Juni 2011

    @stefan… Äh sorry, du hast da ein paar ziemlich falsche Vorstellungen was schwarze Löcher angeht. Dunkle Materie hat nix mit schwarzen Löchern zu tun. https://www.scienceblogs.de/astrodicticum-simplex/2008/12/neptun-vulkan-und-die-dunkle-materie.php

  120. #121 Alderamin
    24. Juni 2011

    Ich glaube die dunkle Materie gibt es nicht, denn man geht davon aus das ein
    Schwarzes Loch einfach so stark ist das es Licht anzieht aber es ist ca. 6 mal
    so Stark (dunkler) als angenommen.

    Dunkle Materie ist ein noch nicht identifizierter Stoff, der die sichtbare Materie begleitet und der an seiner Schwerkraft zu erkennen ist. Es gibt davon ungefähr 4-mal so viel, wie von der gewöhnlichen Materie, aber man hat den oder die Träger der dunklen Materie noch nicht ausfindig machen können.

    Schwarze Löcher sind nicht 6-mal dunkler oder stärker als angenommen, die sind perfekt schwarz, weil ja keine Strahlung an ihnen reflektiert oder von ihnen ausgestrahlt werden kann. Das sind ehemalige Sterne, die so schwer sind, dass an ihrem Lebensende keine Kraft des Universums stark genug wäre, ihren Kollaps unter der eigenen Schwerkraft zu einem winzigen Gebilde zu verhindern, welches dann in unmittelbarer Nähe eine so große Schwerkraft hat, das Licht ihnen nicht entkommen kann. Im Zentrum von Galaxien finden sich sehr massive schwarze Löcher mit Millionen von Sonnenmassen.

    Und das hält die Galaxie zusammen, denke Ich.

    Es gibt zwar anscheinend einen Zusammenhang zwischen der Größe einer Galaxie und der Masse ihres zentralen Schwarzen Lochs, aber die Galaxie würde durch die Schwerkraft ihrer Sterne und der in ihr enthaltenen dunklen Materie auch von allein zusammen halten. Das Schwarze Loch hat nicht mehr Schwerkraft als eine entsprechende Masse von Sternen, und auch Millionen von Sternmassen sind ein Nichts gegenüber der typischen Masse einer Galaxie, die bei hunderten Milliarden Sonnenmassen liegt.

    Ausserdem ist ein Schwarzes Loch so eine art grosser Recycler des Universums
    und die Verbindung in ein anderes Universum, in dem der Kreislauf der Energie
    fortgesetzt wird.

    Nur im Science-Fiction-Roman.

    Jedes grössere Universum Hat ein Schwarzes Loch, weil die dichte irgendwann so
    gross wird das kein Licht mehr nach aussen dringt.

    Wir kennen im Moment nur ein einziges Universum, und das auch nur, so weit wir es überblicken können. Und darin wird die Dichte immer geringer, weil es expandiert, und das immer schneller…

  121. #122 Stefan
    24. Juni 2011

    Sorry, sollte heissen: Jede grössere “Galaxie” hat einschwarzes Loch
    nicht (Universum).
    Alles andere was ich geschrieben habe wird die Wissenschaft vielleicht
    irgendwann beweisen…

  122. #123 Bjoern
    25. Juni 2011

    @Stefan:

    Alles andere was ich geschrieben habe wird die Wissenschaft vielleicht irgendwann beweisen…

    Erstens einmal “beweist” die (experimentelle) Naturwissenschaft sowieso nie etwas (Theorien werden nur belegt, nicht bewiesen). Zweitens wird das Zeug, dass du da oben geschrieben hast, garantiert auch nie belegt – weil es nämlich etabliertem Wissen eklatant widerspricht.

  123. #124 Ralph Ulrich
    25. Juni 2011

    Soweit ich weiss, versucht doch der MOND Ansatz die “dunkle Energie” zu einem Teil wegzukürzen gegen die “dunkle Materie”? Dann hätten wir statt der 75 Prozent “dunkler Energie” nur noch die übriggeblieben 50 Prozent und keine “dunkle Materie”. Wäre ja ästhetischer, weil sparsamer.

    Meine Idee von zusätzlichen an Materie gebundenen Dimensionen hätte zur Folge mit ihren “zusätzlichen Nachbarschaften”, dass Fernwirkung bei der Gravitation besteht auf mittlere Strecken um Galaxien herum. Und dann die Gravitation stark abfällt. Die von der Kürzung übriggebliebene “dunkle Energie” in den Voids (oder auch um Galaxien herum) wirkt mit ihrer Beschleunigung der Gravitation direkt entgegen und verstärkt so dem Anschein nach die Gravitationswirkung in Galaxien wie “dunkle Materie”.

    (Wenn das der MOND Theorie entspricht, ist das nur die zufällige Kovarianz meiner Gedanken…)

  124. #125 Florian Freistetter
    25. Juni 2011

    @Ralph Ulrich: Ne, MOND hat nix mit dunkler Energie zu tun; da gehts nur um dunkle Materie. Da schlägt MOND einfach eine Variation der Gravitationsgleichung vor um die Beobachtung zu erklären. Aber selbst damit kann man nur einen Teil der dunklen Materie verschwinden lassen. Auch MOND kommt nicht ganz ohne dunkle Materie aus.

  125. #126 Bjoern
    25. Juni 2011

    @Ralph Ulrich:

    Soweit ich weiss, versucht doch der MOND Ansatz die “dunkle Energie” zu einem Teil wegzukürzen gegen die “dunkle Materie”?

    Wäre mir neu – und ich wüsste auch nicht, wie das mit MOND gehen sollte. Meines Wissens versucht MOND einfach nur, die dunkle Materie (bzw. einen Teil davon) wegzuerklären.

    Dann hätten wir statt der 75 Prozent “dunkler Energie” nur noch die übriggeblieben 50 Prozent und keine “dunkle Materie”. Wäre ja ästhetischer, weil sparsamer.

    Inwiefern wäre das “sparsamer”?

    Meine Idee von zusätzlichen an Materie gebundenen Dimensionen hätte zur Folge mit ihren “zusätzlichen Nachbarschaften”, dass Fernwirkung bei der Gravitation besteht auf mittlere Strecken um Galaxien herum. Und dann die Gravitation stark abfällt.

    Ich sehe nicht, wie das folgen sollte.

    Die von der Kürzung übriggebliebene “dunkle Energie” in den Voids (oder auch um Galaxien herum) wirkt mit ihrer Beschleunigung der Gravitation direkt entgegen und verstärkt so dem Anschein nach die Gravitationswirkung in Galaxien wie “dunkle Materie”.

    Und auch hier verstehe ich nicht, wie das gehen soll…

  126. #127 Ralph Ulrich
    25. Juni 2011

    Verbesserung: “wirkt mit ihrer Beschleunigung der Gravitation direkt entgegen”
    Wollte sagen: “dunkle Energie” ist eine Kraft mit dem umgekehrten Vorzeichen der Gravitation.

  127. #128 Bjoern
    25. Juni 2011

    @Ralph Ulrich:

    Wollte sagen: “dunkle Energie” ist eine Kraft mit dem umgekehrten Vorzeichen der Gravitation.

    Also, erstens einmal ist eine Energie etwas anderes als eine Kraft (Grundlagen Physik, ca. 8. Klasse…), man könnte also höchstens sagen “dunkle Energie bewirkt eine Kraft mit…” oder so. Zweitens wäre selbst letztere Aussage schlichtweg falsch bzw. sinnlos. Die Dunkle Energie ist homogen verteilt – in welche Richtung sollte die Kraft also zeigen?!?

  128. #129 Ralph Ulrich
    25. Juni 2011

    @Bjoern, wenn du einen Gravitationspunkt hast, und aussen herum sind Gebiete, die anti-gravitativ wirken, dann sieht es aus, als sei die Gravitation des Gravitationspunktes verstärkt (“dunkle Materie”).

  129. #130 Bjoern
    25. Juni 2011

    @Ralph Ulrich: Was soll ein “Gravitationspunkt” sein?

  130. #131 Ralph Ulrich
    25. Juni 2011

    @Bjoern, ja meine physikalisch nicht trainierte Ausdrucksweise ….

    Bjoern: “Die Dunkle Energie ist homogen verteilt – in welche Richtung sollte die Kraft also zeigen?”
    Das ist ja meine Idee, dass diese “dunkle Energie” eben nicht genau homogen verteilt ist, sondern vom Grad der Reinheit des Vakuums abhängt! Daruüber spreche ich doch die ganze Zeit schon !!!

    Soweit ich weiss, sagen die Messergebnise nur, dass die “dunkle Energie” konstant in Bezug auf alle Richtungen ist.

  131. #132 Ralph Ulrich
    25. Juni 2011

    @Bjoern: “Was soll ein “Gravitationspunkt” sein?”
    Massepunkt sagt man, oder wie?

  132. #133 Bjoern
    25. Juni 2011

    @Ralph Ulrich:

    Das ist ja meine Idee, dass diese “dunkle Energie” eben nicht genau homogen verteilt ist, …

    Tja, die Beobachtungen sagen aber was anderes.

    Vorschlag: schau’ dir doch einfach mal an, welche Beobachtungen überhaupt dazu geführt haben, dass man die Dunkle Energie eingeführt hat, und versuch’ diese Beobachtungen dann erst mal mit deinen Ideen zu erklären, bevor du versuchst, damit auch noch die dunkle Materie weg zu erklären…

    …sondern vom Grad der Reinheit des Vakuums abhängt! Daruüber spreche ich doch die ganze Zeit schon !!!

    Ach so? Also, irgendwie schafft du es wirklich beachtlich schlecht, klarzumachen, was du eigentlich meinst…

    Soweit ich weiss, sagen die Messergebnise nur, dass die “dunkle Energie” konstant in Bezug auf alle Richtungen ist.

    Dann weisst du falsch.

  133. #134 Alderamin
    25. Juni 2011

    @Ralph

    Das ist ja meine Idee, dass diese “dunkle Energie” eben nicht genau homogen verteilt ist, sondern vom Grad der Reinheit des Vakuums abhängt! Daruüber spreche ich doch die ganze Zeit schon !!!

    Na, aber sie isses doch, jedenfalls habe ich das schon mehrfach in Sky&Telescope gelesen. Sie ist eine kosmologische Konstante. Endegelände.

  134. #135 Ralph Ulrich
    27. Juni 2011

    Es gibt, wie ich mit Sicherheit vermutete, tatsächlich Mathematiker, die sich Gedanken über die Gravitation machen per Geometrie. Ob das allerdings meinen Gedanken entspricht, kann ich nicht beurteilen:
    https://resonaances.blogspot.com/2007/02/alain-connes-standard-model.html

    His program consists in identifying a structure of space-time could give rise to the Standard Model + gravity. He finds the answer is the product of an ordinary spin manifold by a finite noncommutative discrete space of KO-dimension 6 modulo 8 and of metric dimension 0, whatever it means. The discrete space is responsible for the spectrum, symmetries and the interactions of the Standard Model. Most of the Standard Model parameters correspond to the freedom of parametrizing the internal geometry.

  135. #136 Bjoern
    27. Juni 2011

    @Ralph Ulrich:

    Es gibt, wie ich mit Sicherheit vermutete, tatsächlich Mathematiker, die sich Gedanken über die Gravitation machen per Geometrie. Ob das allerdings meinen Gedanken entspricht, kann ich nicht beurteilen:

    Gravitation durch Geometrie beschreiben macht bereits die Allgemeine Relativitätstheorie (das sag’ ich hier auch nicht zum ersten Male…) Das neue an Connes’ Ideen ist, dass er versucht, auch die Elementarteilchenphysik auf eine rein geometrische Grundlage zu stellen – nicht (nur) die Gravitation!

  136. #137 Ralph Ulrich
    27. Juni 2011

    @Bjoern, soviel hatte ich auch verstanden, und dann hat es mich natürlich interessiert, weil ich den gravitativen Effekt aus dem geometrischen Aufbau von Materie ableite. Aber wie ich an der kurzen Erklärung auf der gelinkten Blog Seite gesehen habe, werde ich Connes’ Theorie nicht verstehen: Angeblich standen ja auch die Cern Mitarbeiter wie Ochs vorm Berg bei Connes Begriffen, und das sind studierte Phyiker gewesen mit mehr mathetischen Grundlagen als ich.

  137. #138 kraxn
    26. August 2011

    seid bitte nachsichtig, ich kenn mich wirklich nicht aus, und spiele so einige Gedanken durch.

    so weit ich es verstanden habe können schwarze Löcher “wachsen”.

    Wäre es möglich, dass ein schwarzes Loch ein anderes “frisst”?

    Wäre es denkbar, dass in -zig hundert milliarden Jahren durch das gegenseitige fressen und Wachsen der schwarzen löcher nix mehr überbleibt als ein riesig großes schwarzes Loch.

    LG
    kraxn

  138. #139 rolak
    26. August 2011

    /soweit ich es…/ ja
    /wäre es möglich…/ ja, obgleich ‘fressen’ nicht besonders passend ist
    /wäre es denkbar…/ jein, siehe da hinten

  139. #140 Alderamin
    26. August 2011

    @kraxn

    Mein Senf auch noch dazu 😉

    so weit ich es verstanden habe können schwarze Löcher “wachsen”.

    Ja, je mehr Masse ein schwarzes Loch schluckt, desto größer wird es, jedenfalls der Bereich, dem Licht nicht mehr entkommen kann (Schwarzschild-Radius)

    Wäre es möglich, dass ein schwarzes Loch ein anderes “frisst”?

    Wenn sie sich nahe genug kommen, ja. Nur ist es nicht so (was dann die Antwort die letzte Frage ist), dass schwarze Löcher alles staubsaugermäßig ansaugen, sondern sie haben zunächst mal exakt die selbe Schwerkraft, wie die Masse verusacht, die sie verschluckt haben. Ein schwarzes Loch von Sonnenmasse hat genau die Anziehungskraft der Sonne. Da es aber viel kleiner als die Sonne wäre, könnte man viel näher an den Schwerpunkt heran, und daher wäre dort die Anziehungskraft viel größer, als auf der Oberfläche der Sonne.

    Damit zwei schwarze Löcher verschmelzen können, müssen sie sich stark annähern. Das kann auf zweierlei Weisen geschehen:

    1) Bei stellaren Schwarzen Löchern, die schon als Sterne sehr nahe umeinander kreisten, werden beim Umkreisen Gravitationswellen ausgesendet, die Energie forttragen. Dadurch rücken die schwarzen Löcher langsam aufeinander zu, bis sie irgendwann kollidieren und verschmelzen.

    2) Wenn zwei Galaxien verschmelzen, enthalten diese auch schwarze Löcher. In der Wechselwirkung mit anderen Sternen tauschen sie über die Gravitation Bewegungsenergie aus und neigen dazu, auf diese Weise nach innen zu sinken. Dort können sie sich dann irgendwann begegnen, in Umlauf umeinander geraten (direkte Treffer sind nahezu unmöglich) und dann nach dem in 1) genannten Prozess schließlich verschmelzen.

    Wäre es denkbar, dass in -zig hundert milliarden Jahren durch das gegenseitige fressen und Wachsen der schwarzen löcher nix mehr überbleibt als ein riesig großes schwarzes Loch.

    Nein, denn durch das Zusammenrücken der Schwarzen Löcher bei verschmelzenden Galaxien werden dafür andere Sterne weiter nach außen “in Sicherheit” befördert. Und es verschmelzen auch nicht alle Galaxien, sondern der überwiegende Teil entfernt sich voneinander. Nur große Sterne ab 10 Sonnenmassen und die Zentren von Galaxien werden zu Schwarzen Löchern, die dann durch die nach Stephen Hawking benannte Hawking-Strahlung ganz allmählich wieder zerfallen. Aber das dauert extrem lange.

  140. #141 kraxn
    26. August 2011

    Wow, danke für die schnellen Antworten.

    Das alles ist wirklich unheimlich spannend und fazinierend.

    ….und wieder etwas gelernt

    LG
    Kraxn

  141. #142 Carsten Czarski
    49448 Lemförde
    22. Mai 2015

    Meine Frage ist: Wenn ich doch durch ein Teleskop schaue, dann sagt man das es ein paar tausend Lichtjahre weg ist. Soweit klar. Doch ich schaue der Zeit ja entgegen, weil ich das Licht ja früher sehen kann als ohne Teleskop.
    So müsste es doch auch mit dem Hubble und Wise sein, oder verstehe ich da was nicht? Okay- das wollte ich mal gefragt haben. Besten Gruß

  142. #144 Alderamin
    22. Mai 2015

    @Carsten Czarski

    weil ich das Licht ja früher sehen kann als ohne Teleskop.

    Nö, das Licht durch das Teleskop ist nicht schneller im Auge als ohne. Was Zeit kostet, ist das Zurücklegen der Strecke vom fernen Stern bis zum Auge, egal ob das Licht direkt ins Auge fällt, oder durch ein Teleskop.

    Das Teleskop sammelt nur mehr Licht und stellt das Objekt größer dar, als mit bloßem Auge.